You are on page 1of 25

Pediatra

Desgloses
comentados
497
Desgloses comentados
T1
Neonatologa
P141 MIR 2011-2012
La enfermedad de las membranas hialinas o sndrome de distrs respi-
ratorio del recin nacido prematuro es debida a la inmadurez pulmonar
por dfcit de surfactante pulmonar. ste, fabricado por los neumocitos
tipo II a partir de las 20-24 semanas de edad gestacional, es un agen-
te tensioactivo que permite la apertura de los alvelos y, por tanto, un
adecuado intercambio gaseoso. Est formado por protenas, colesterol,
fosfatidilcolina, esfngomielina y, fundamentalmente, por lecitina, cuya
cantidad va aumentando segn avanza el embarazo. La madurez pulmo-
nar se alcanza fnalmente a las 34-35 semanas de edad gestacional, de
manera que por debajo de esta edad gestacional, la falta de surfactante
provoca un aumento de la tensin superfcial que determina el colapso
alveolar y un shunt intrapulmonar (alvelos perfundidos, pero no ventila-
dos). Clnicamente, observaremos un pretrmino con signos de difcultad
respiratoria en las primeras horas de vida, con cianosis, mala respuesta al
oxgeno, hipercapnia, acidosis respiratoria y crepitantes a la auscultacin.
En la radiografa de trax observamos un parnquima retculo nodular
fno, tambin denominado patrn en vidrio esmerilado, con broncogra-
ma areo y escaso volumen pulmonar.
El tratamiento de eleccin es la administracin de surfactante exge-
no por va endotraqueal, garantizando adems un soporte respiratorio
adecuado, evitando la hiperoxia, que es mal tolerada por el pretrmino
(uno de los principales factores de riesgo para desarrollar retinopata de
prematuro). La principal complicacin a largo plazo es la evolucin a una
displasia broncopulmonar. La administracin de corticoides prenatales a
la madre, ante la amenaza de parto prematuro, disminuye su incidencia
y su gravedad.
P179 MIR 2008-2009
La cocana no suele producir sndrome de abstinencia en el recin nacido, a
diferencia de los opiceos. Sin embargo, pueden aparecer complicaciones
obsttricas, como el parto pretrmino, el desprendimiento prematuro de
placenta, etc.
Por otra parte, puede afectar directamente al feto, manifestndose como
CIR, microcefalia, hemorragias intracraneales, anomalas digestivas o rena-
les, muerte sbita, alteraciones conductuales o neurolgicas.
La respuesta correcta, por tanto, es la 3. Lo que produce la cocana no es
parto postrmino, sino todo lo contrario: pretrmino.
P184 MIR 2008-2009
Hoy en da, es poco frecuente diagnosticar clnicamente un hipotiroidismo
congnito debido a que, en su mayor parte, son diagnosticados con el cri-
bado neonatal, que se realiza determinando los niveles de TSH obtenida
entre los dos y los cinco das de vida (cuyos valores estaran elevados).
Las manifestaciones del hipotiroidismo congnito tardan varias semanas en
presentarse plenamente, estando totalmente establecidas hacia el 3-6 mes
de vida. Aparece una facies peculiar (cara tosca, prpados y labios tumefactos,
nariz corta con base deprimida, hipertelorismo, boca semiabierta y macroglo-
sia). Otras manifestaciones tpicas son el retraso en la maduracin sea, la her-
nia umbilical, la ictericia prolongada, la letargia y el estreimiento.
El principal problema del hipotiroidismo congnito es su repercusin so-
bre el desarrollo intelectual. ste sera ms grave cuanto ms se demore el
diagnstico (respuesta 3 correcta), por lo que es importante instaurar el
tratamiento cuanto antes. Aunque la causa ms frecuente de hipotiroidis-
mo congnito es la disgenesia tiroidea, independientemente de cul sea la
causa exacta, ante cualquier caso de hipotiroidismo debemos apresurar-
nos a establecer el tratamiento sustitutivo lo antes posible.
P187 MIR 2008-2009
Un caso clnico complicado, donde resulta bastante difcil decidirse entre
las opciones 2 y 5. Las otras son relativamente fciles de descartar, por las
razones siguientes:
R1: no existen signos de hemorragia intraventricular, que normalmen-
te cursara con bradicardia, hipoventilacin, convulsiones, hipotona y
fontanelas a tensin (slo coincide la hipotona, que es un dato ines-
pecfco).
R3: la fenilcetonuria es una enfermedad metablica en la que se pro-
duce una acumulacin anormal de fenilalanina. Se manifesta varias
semanas despus del nacimiento. La mayora de estos enfermos mues-
tran piel clara, ojos azules y color claro del pelo (no puede sintetizarse
la melanina, ya que la fenilalanina es un precursor suyo y no puede
metabolizarse en el hgado). Es tpico el retraso mental por acmulo
de fenilalanina en plasma hasta 30 veces por encima del nivel normal.
R4: la enfermedad de la membrana hialina producira una imagen de
condensacin pulmonar, con broncograma areo y habra alteracio-
nes en la auscultacin pulmonar (en este caso, nos dicen que ventila
sin problemas).
La verdadera difcultad estara en distinguir la hipoglucemia de la sepsis
neonatal. Al ser hijo de madre diabtica tiene un claro factor de riesgo para
padecer una hipoglucemia. Sin embargo, al ser un recin nacido pretr-
mino, tambin lo tiene para sepsis. Por tanto, debemos guiarnos por las
manifestaciones clnicas.
498 Desgloses comentados
Pediatra
La presencia de acrocianosis y malestar general, as como el color terroso,
podran aparecer en ambos casos. Sin embargo, cuando se trata de una
hipoglucemia, lo habitual sera la aparicin de temblores o algn otro tipo
de clnica neurolgica (irritabilidad, convulsiones) que en este caso faltan.
Como en esta ocasin predominan las manifestaciones inespecfcas, sin
foco infeccioso claramente identifcable, resulta ms probable la respuesta
2 que la 5.
P185 MIR 2007-2008
Pregunta asequible acerca del estudio de infecciones connatales en el re-
cin nacido, si bien haca aos que no aparecan en el MIR.
Las tres primeras opciones tocan aspectos generales del manejo de estas
infecciones. En todas, es til el estudio serolgico del RN (opcin 2 correc-
ta), especialmente de la IgM especfca al germen, ya que es de sntesis
fetal (recuerda que es pentamrica y que por sus dimensiones no cruza
placenta).
Puedes repasar todo lo comentado en la siguiente tabla.
P185 (MIR 07-08) Infecciones connatales
El estudio placentario (opcin 1 correcta), a nivel histolgico, puede mos-
trar datos como amnionitis, infartos, hipervascularizacin, abruptio
E incluso puede cultivarse. Muchas veces, la clnica no es especfca y se
manifesta como CIR, citopenias, ictericia precoz, e incluso de forma asinto-
mtica (opciones 3 y 4 correctas). Este estado asintomtico puede acarrear
secuelas irreversibles. En concreto, la forma de presentacin ms frecuente
de la toxoplasmosis es una coriorretinitis silente que puede abocar a la ce-
guera en la edad adulta. Por ello, todos los recin nacidos infectados han
de ser tratados con sulfadiacina, pirimetamina y cido folnico durante el
primer ao de vida (opcin 5 falsa).
Asimismo, fjate que las respuestas 1, 3 y 4 tienen un puede. En Medicina,
ya sabes que nada es seguro. Es muy difcil que unas opciones tan poco
categricas resulten falsas.
P181 MIR 2005-2006
Pregunta de difcultad moderada acerca del retraso en la eliminacin del
meconio.
En condiciones normales, el RN expulsa el meconio en las primeras 24-36
horas de vida. Si la eliminacin de meconio no se produce, se acumula una
masa compacta a escala anorrectal que impide la expulsin. Son causas de
retraso en la eliminacin de meconio:
1. El sndrome de colon izquierdo hipoplsico (frecuente en hijos de ma-
dre diabtica).
2. Fibrosis qustica.
3. Aganglionosis rectal.
4. Drogadiccin materna.
5. Prematuridad.
6. Tratamiento con sulfato de magnesio de la preeclampsia materna.
El hipotiroidismo congnito no cursa en principio con retraso en la elimina-
cin del meconio, porque la clnica de instauracin es tarda, estando com-
pletamente establecida a los tres o seis meses de edad. Son datos sugestivos:
una facies peculiar (cara tosca con prpados y labios tumefactos, nariz corta
con base deprimida, hipertelorismo con boca abierta y macroglosia), estrei-
miento, hernia umbilical, ictericia prolongada, letargia y un retraso en la madu-
racin sea. Sin embargo, hoy en da el diagnstico clnico es poco corriente
gracias a las pruebas de screening precoz mediante la determinacin de la TSH
en muestra de sangre obtenida a las 48 h y a los cinco das de vida de todo RN.
Unos niveles de TSH menores de 10 mcUI/ml son compatibles con la norma-
lidad, mientras que si la TSH supera los 50 mcUI/ml el caso es compatible con
un hipotiroidismo primario congnito y debe instaurarse precozmente trata-
miento con levotiroxina con el objetivo de mejorar el pronstico neurolgico
de estos pacientes. La causa ms frecuente de hipotiroidismo congnito es la
disgenesia tiroidea, que incluye tiroides aplsicos, hipoplsicos y ectpicos,
siendo esta ltima la causa ms comn de hipotiroidismo congnito.
En la primera plantilla de respuestas que dio el Ministerio se considera-
ba como respuesta correcta la opcin 5; no obstante, posteriormente la
pregunta fue anulada, ya que algunos autores consideran que el hipotiroi-
dismo congnito puede estar tambin implicado en el retraso en la elimi-
nacin del meconio.
P168 MIR 2004-2005
Pregunta difcil sobre los efectos de los corticoides. La que no debe plan-
tearte ningn tipo de duda es la 5, puesto que se indican con este fn, ace-
lerar la sntesis pulmonar de surfactante. Adems, reducen la incidencia de
499
Desgloses comentados
Pediatra
hemorragia de la matriz germinal (respuesta 3 correcta), de NEC (respuesta
4 correcta), de DAP y de neumotrax.El resto de las opciones no deben
preocuparte.
La opcin 1 es falsa, porque los corticoides nicamente aumentan el riesgo
de infeccin cuando se emplean crnicamente, produciendo en tal caso
inmunodepresin celular. Pero el uso puntual de corticoides, aunque sea
una dosis relativamente alta, carece de este efecto.
P170 MIR 2004-2005
La incompatibilidad Rh requiere que una madre, Rh negativa, sea
puesta en contacto previamente con sangre Rh positiva. En cambio,
la incompatibilidad ABO puede afectar al primer hijo sin necesidad de
sensibilizacin previa (respuesta 1 correcta). Esta circunstancia es ms
frecuente (respuesta 5 falsa), pero menos grave, que la incompatibili-
dad Rh, por lo que no se asocia a anemias fetales severas (respuestas
2 y 3 falsas).
El test de Coombs indirecto es positivo y el directo tambin suele serlo,
salvo en raras excepciones, de modo que la opcin 4 tambin se podra
considerar correcta. En consecuencia, estamos ante una pregunta que po-
dra impugnarse.
P170 (MIR 04-05) Isoinmunizacin del recin nacido (RN)
P192 MIR 2004-2005
La enterocolitis necrotizante suelen presentarla en el MIR como un cua-
dro de distensin abdominal y deposiciones sanguinolentas. Sin em-
bargo, esta pregunta va un poco ms all. Observa que nos dan tres
antecedentes que implicaran un mayor riesgo para esta enfermedad:
prematuridad, bajo peso, necesidad de ventilacin asistida... Si a esto
aadimos un cuadro clnico compatible con obstruccin intestinal (es-
treimiento, vmitos, distensin y dolor clico), la opcin que mejor
encaja es la 1.
De hecho, una de las complicaciones ms habituales de esta entidad
son las estenosis intestinales, que generan cuadros suboclusivos, como
en este lactante que nos comentan.
P192 (MIR 04-05) Enterocolitis necrotizante. Radiologa
P168 MIR 2003-2004
Pregunta de difcultad moderada. Teniendo en cuenta la clnica descrita en
el caso sera razonable dudar entre las dos ltimas opciones, pero conside-
rando los antecedentes expuestos, no queda opcin a duda.
La vitamina E es una vitamina liposoluble cuya defciencia aparece unida
a enfermedades malabsortivas graves y prolongadas: fbrosis qustica, co-
lestasis (y dentro de stas la atresia de vas biliares), etc. La carencia nutri-
cional no se ha descrito. Se manifesta por un sndrome potencialmente re-
versible caracterizado por ataxia (cerebelosa y por afectacin de cordones
posteriores) y neuropata perifrica.
Con respecto al resto de opciones, es interesante comentar la 5. Esta pa-
tologa debuta en la infancia o adolescencia con ataxia progresiva de la
marcha y, ya que su diagnstico es eminentemente clnico (apoyndonos
en los antecedentes familiares, EMG, etc.), es fundamental descartar pre-
viamente un dfcit de vitamina E.
Las opciones 1 y 3 son incorrectas, puesto que en el caso no se describe nin-
guna alteracin del estado mental. El dfcit de vitamina A cursa con ceguera
nocturna, xeroftalmia, retraso del crecimiento, apata, sequedad cutnea, etc.
Como comentario fnal resear la importancia de la reposicin peridica
de vitaminas liposolubles en las colestasis neonatales.
500 Desgloses comentados
Pediatra
P191 MIR 2002-2003
Pregunta fcil sobre el distrs respiratorio del recin nacido.
Mediante este caso clnico se nos exponen las principales caractersti-
cas de lo que se conoce como: sndrome de Avery, pulmn humedo,
taquipnea transitoria del recin nacido o sndrome de distrs respira-
torio tipo II.
Epidemiologa: recin nacidos a trmino o postrmino por cesrea o
por parto vaginal muy rpido.
Patogenia: retraso en la absorcin del lquido de los pulmones.
Clnica: distrs respiratorio que se valora mediante el test de Silver-
man (regla mnemotcnica: DIRE QUE ATLETIC: DI-sociacin trax-
abdomen; RE-traccin xifoidea; QUE-jido respiratorio; ALE-teo nasal y
TI-raje). Mejora con la administracin de oxgeno.
Diagnstico: Rx de trax: lquido en las cisuras, hiperinsufacin, dia-
fragmas aplanados, derrame pleural, no broncograma; GAB: normal.
Tratamiento: atmsfera enriquecida en oxgeno, recuperacin en uno o
dos das. Por eso, la opcin correcta es la 1. La opcin 2 es el tratamiento
de la enfermedad de membrana hialina o sndrome de distrs respirato-
rio tipo I (con la cual se podra plantear duda, porque es tpico de recin
nacidos pretrmino, pero su imagen radiolgica tpica es la afectacin
bibasal con infltrado retculo-nodular y broncograma).
P192 MIR 2002-2003
Pregunta de difcultad media sobre un tema que es muy preguntado,
como es la enterocolitis necrotizante.
La enterocolitis necrotizante es una lesin isqumico-necrtica que suele
afectar a leon distal y colon proximal y que con frecuencia se acompaa de
sepsis bacteriana (opcin 1 incorrecta).
Etiologa: es tpica de recin nacidos de muy bajo peso con enfer-
medades graves, policitemia, alimentacin muy precoz y con elevados
volmenes, hipoxia... No obstante, parece que la lactancia materna
puede ser un factor protector (tambin lo es de la retinopata de la
prematuridad) (opcin 3 correcta). Se ha involucrado a diversos pat-
genos (E. coli, C. perfringens, rotavirus y S. epidermidis), pero en la ma-
yora de los casos no se detecta ningn germen responsable. (opcin
2 incorrecta).
Clnica: repentina distensin abdominal, retencin gstrica y deposi-
ciones sanguinolentas en la 2 semana.
Diagnstico: laboratorio: leucopenia, neutropenia, trombopenia (op-
cin 4 incorrecta), hiponatremia. Rx de abdomen: signos ms precoces
son el edema de asas y el asa fja; patrn en miga de pan, neumatosis
intestinal (signo diagnstico), neumoperitoneo (si hay perforacin) y
gas en vena porta.
Tratamiento: dieta absoluta, fuidoterapia y descompresin nasogs-
trica; antibiticos; tratamiento quirrgico si hay perforacin, peritoni-
tis o sepsis refractaria al tratamiento mdico.
P177 MIR 2001-2002
Existen mltiples causas de difcultad respiratoria en el neonato.
La taquipnea transitoria del recin nacido se produce por un retraso en
la absorcin del lquido de los pulmones fetales. Aparece en RNT o RNPT
nacidos por cesrea o parto vaginal rpido. Clnicamente, el neonato pre-
senta distrs respiratorio leve-moderado, de inicio precoz y que mejora
con pequeas cantidades de oxgeno. La gasometra no suele mostrar
alteraciones signifcativas y en la radiografa se observa un aumento de
las marcas vasculares, lquido en las cisuras, aplanamiento de los diafrag-
mas y, a veces, derrame pleural. No existe broncograma areo (respuesta
4 correcta).
La enfermedad de la membrana hialina (EMH) se produce por un d-
fcit de surfactante pulmonar, lo que provoca un aumento de la tensin
superfcial y colapso de los alveolos. Afecta, sobre todo, a RNPT (dado que
los niveles de surfactante no alcanzan la madurez pulmonar hasta las 35
semanas de edad gestacional). Cursa con un distrs respiratorio precoz
moderado-severo, con cianosis progresiva que con frecuencia no mejora
con la oxigenoterapia.
En la gasometra hay hipoxemia, hipercapnia y acidosis metablica y/o
respiratoria, y en la radiografa es tpico (pese a que no patognomnico,
pues una radiografa similar se puede ver en la sepsis neonatal) un infltra-
do reticulogranular con broncograma areo.
El sndrome de aspiracin de meconio (SAM) es un cuadro tpico de RN
postrmino que ha padecido un sufrimiento fetal agudo. La gravedad cl-
nica depende de lo espeso del meconio. El RN presenta distrs respirato-
rio y signos de atrapamiento areo, con hiperinsufacin torcica y riesgo
de extravasacin extrapulmonar de aire, dando como complicaciones
neumotrax, neumomediastino o neumopericardio.
El neumomediastino suele ser asintomtico, apareciendo en algunos ca-
sos una distensin de las venas del cuello, por estar difcultado el retorno
venoso. Los tonos cardacos estn atenuados.
La radiografa tpica del SAM es un infltrado parcheado, de aspecto al-
godonoso, con diafragmas aplanados. A veces tambin neumotrax o
neumomediastino.
El sndrome de Wilson-Mikity es un cuadro clnico radiolgicamente similar
a la displasia broncopulmonar (cuadro producido al someter a neonatos a
ciertos procedimientos teraputicos, como la oxigenoterapia a altas concen-
traciones) que se describi en RNPT sin antecedentes de EMH.
P178 MIR 2001-2002
La meningitis en el recin nacido puede estar causada por bacterias, virus,
hongos o protozoos. La meningitis puede presentarse dentro de una sepsis o
como una infeccin focal. Las causas bacterianas ms normales de meningitis
neonatal son los estreptococos del grupo B, E. coli y Listeria. Tambin pueden
producir meningitis otros estreptococos, cepas no tipifcables de H. infuenzae
y estaflococos, entre otros.
P178 (MIR 01-02) Sepsis y meningitis neonatal
501
Desgloses comentados
Pediatra
La duda razonable est entre las opciones 4 y 5, dado que ambos pue-
den causar meningitis neonatal. Sin embargo, el estafilococo es uno
de los grmenes implicados en la sepsis de origen nosocomial, sobre
todo en neonatos que han estado cateterizados. El cuadro clnico con-
siste inicialmente en una sepsis sin foco, apareciendo con frecuencia,
ms tarde, focalidad menngea (respuesta 4 incorrecta; respuesta 5
correcta).
Las manifestaciones clnicas de la meningitis neonatal pueden ser inicialmen-
te inespecfcas, con o sin signos neurolgicos (letargia, fontanela abomba-
da, convulsiones...). El diagnstico se confrma mediante el examen del LCR
y la identifcacin del microorganismo. El LCR se considera patolgico si hay
pleocitosis (ms de 30 clulas/mm
3
, con polimorfonucleares mayor del 60 %),
hiperproteinorraquia (mayor de 150 mg/100 ml) e hipoglucorraquia (glucosa
menor de 30 mg/dl).El tratamiento de la meningitis del RN no ingresado se
establece empricamente con ampicilina ms cefotaxima, y en el RN ingresado
con vancomicina ms ceftacidima +/ anfotericina B.
P185 MIR 2001-2002
El trasplante heptico representa el tratamiento convencional de las he-
patopatas terminales de la infancia. La indicacin ms frecuente en los
nios es la atresia biliar extraheptica que no responde a la portoente-
rostoma de Kasai. Las enfermedades metablicas del hgado, entre ellas el
dfcit de alfa 1- antitripsina, que es la ms usual, y la colestasis familiar le
siguen en frecuencia.
En los centros de trasplante se utilizan varios sistemas de puntuacin para
predecir la mortalidad por hepatopata, pero los factores ms destacados
son la prolongacin del tiempo de tromboplastina parcial, la ascitis, el au-
mento de los niveles de bilirrubina indirecta y la disminucin del coleste-
rol. Algunos trastornos, como la atresia biliar sin respuesta a la ciruga de
Kasai, deben corregirse con trasplante antes de que aparezcan complica-
ciones tardas.
El tratamiento previo al trasplante resulta decisivo para el xito de la inter-
vencin. Los aspectos ms relevantes comprenden la nutricin, las vitami-
nas, la vacunacin y la atencin sanitaria general del nio.
El tratamiento mdico tambin comprende el control de la hipertensin
portal. El estado del paciente en el momento del trasplante, a excepcin de
los casos de coma profundo, no infuye en la supervivencia, pero s afecta
al perodo de recuperacin y a las complicaciones. El pronstico para los
supervivientes es muy favorable. El crecimiento mejora y los estigmas de
hepatopata crnica desaparecen.
Repasa las indicaciones de trasplante con la siguietne tabla.
P177 (MIR 01-02) Distrs respiratorio
502 Desgloses comentados
Pediatra
P185 (MIR 01-02) Indicaciones del trasplante de hgado en la infancia
Indicaciones N de casos
Atresia biliar 145
Enfermedad metablica del hgado 42
- Dficit de -1 antitripsina 27
- Tirosinemia 7
- Enfermedad de Wilson 4
- Otras 4
Colestasis familiar 19
Insuficiencia heptica fulminante 16
Hipoplasia biliar 13
Hepatitis 12
Cirrosis idioptica 7
Hepatitis neonatal 5
Colangitis esclerosante 3
Fibrosis heptica congnita 2
Carcinoma 2
Miscelanea 4
Total 270
De Ziell BJ, Gartner JC, Malatack JJ, y cols.: liver transplantation in children: A
pediatricians perspective.
Pediatr Ann 20:691,1191.

T2
Desarrollo y nutricin
P152 MIR 2010-2011
Pregunta sobre el desarrollo psicomotor del nio. En Pediatra son tpicas estas
preguntas, son bastante difciles y especfcas, ya que los periodos que nos pro-
ponen son muy concretos, y existen diversos hitos del crecimiento y desarrollo
que pueden solaparse en diferentes momentos del tiempo.
La prueba ms empleada es el test de Denver, que valora los aspectos per-
sonal, social, lenguaje y motricidad fna y grosera en los nios desde el na-
cimiento hasta los 6 aos.
La respuesta correcta es la opcin 3. Corre, sube escaleras andando cogido
de la mano, construye una torre de cuatro cubos, se sienta en sillas peque-
as, dice 10 palabras (de media), identifca una o ms partes del cuerpo,
come solo, pide ayuda en caso de necesitarla, etc.
P166 MIR 2009-2010
Una pregunta de difcultad media que se responde de forma directa a par-
tir del Manual CTO.
Tal como explicamos en el Captulo 2 (Desarrollo y nutricin) de la Seccin
de Pediatra, el indicador ms sensible para detectar precozmente este tipo
de alteraciones es la velocidad de crecimiento (respuesta 2 correcta). sta
es mxima durante los cuatro primeros aos de la vida, producindose un
segundo incremento (aunque menor) durante la pubertad, siendo ms
precoz en nias que en nios.
Los datos que hay que tener en cuenta para evaluar una talla baja son la
velocidad de crecimiento y la correlacin con la talla de los padres. En este
caso clnico se trata de un nio con una velocidad de crecimiento normal.
Por otra parte, la talla se puede considerar baja pero dentro de los lmites
normales y, sobre todo, acorde con la talla materna.
Adems, nos explican que en la rama paterna, existen antecedentes de pu-
bertad retrasada.
Por consiguiente, no est indicado utilizar hormona del crecimiento ni
realizar estudios del eje hipotlamo-hipofsario. Tampoco cabra esperar lo
que dice la respuesta 3 (de hecho, sera al contrario: edad sea retrasada
con respecto a la edad cronolgica). En otras palabras, no estamos ante un
caso en el que haya que pensar en patologa de ningn tipo, al menos con
estos datos, por lo que no precisa que tomemos medidas extraordinarias
P188 MIR 2008-2009
Una pregunta aparentemente difcil sobre el desarrollo psicomotor que re-
sulta mucho ms sencilla si la analizas con lgica.
La duda razonable estara entre las opciones 2 y 3. De hecho, en esta con-
vocatoria, a la mayor parte de los opositores les sonaba la idea de los seis
meses. La difcultad est en decidirse por una u otra. En el Manual CTO, la
solucin apareca claramente, y citamos de forma textual: 6 mes: inicia la
sedestacin, que se completa a los 8 meses. Por tanto, si la sedestacin se
inicia en el 6 mes, difcilmente habr un 50% de los lactantes que ya puedan
sentarse antes de ese momento. La respuesta correcta sera entonces la 3.
Por otra parte, ante una duda como sta, es posible que, en el momento
del examen, no dispongamos de ningn argumento terico y haya que
arriesgar entre dos opciones. En tal caso, ante una pregunta cuyas cinco
opciones son numricas, la apuesta ms segura sera el valor intermedio,
ordenndolos todos de menor a mayor. En realidad, as es como muchos
opositores ganaron el punto de esta pregunta.
P181 MIR 2007-2008
El crecimiento y desarrollo en pediatra es un tema complejo debido a su
amplitud. Para el MIR, solamente es necesario conocer un pequeo resumen.
Esta pregunta se anul. Durante la primera semana de vida el recin nacido
(RN) puede sufrir una prdida de peso fsiolgica en torno al 5-10% con
respecto al nacimiento, resultado de la excrecin del exceso del lquido ex-
travascular y la ingesta escasa. Por eso, la opcin 2 es falsa, puesto que no
suele disminuir ms del 10% (prdida de peso patolgica).
A medida que la madre mejora en la tcnica de la lactancia y el calostro es
reemplazado por una leche ms rica en grasas, el RN recupera peso, alrededor
de 30 g/da durante el primer mes (opcin 3 correcta). As, al fnal del primer
ao de vida, el peso al nacimiento aproximadamente se triplica y la talla se
incrementa un 50% con respecto al nacimiento (si al nacer mide 50 cm, al fnal
del primer ao medira 75 cm). Por ello, la opcin 1 sera tambin falsa, dado
que la duplicacin de la talla ocurre alrededor de los cuatro aos de vida.
Las opciones 4 y 5 son correctas y hacen referencia al desarrollo psicomo-
tor. Como norma, la mayora de los refejos arcaicos desaparecen en torno
a los cuatro meses de vida. No olvides que el desarrollo psicomotor sigue
un patrn cefalocaudal, y uno de los primeros hitos que aparecen es la son-
risa social hacia el 1,5-2 meses de vida.
503
Desgloses comentados
Pediatra
P190 MIR 2007-2008
Una pregunta relativamente sencilla sobre la talla baja, ya que el sndrome
de Klinefelter produce precisamente lo contrario: talla alta, sobre todo a
expensas de la mitad inferior del cuerpo.
Para acertar esta pregunta, en realidad no era necesario conocer rarezas
como las que enuncian las respuestas 3 y 4. El sndrome de Seckel, entre
otras manifestaciones, ocasiona microcefalia, bajo peso al nacer y talla
baja. El de Silver-Russell, que es causa de talla baja, tambin se asocia con
malformaciones esquelticas (asimetra) y alteraciones morfolgicas del
crneo. Los sndromes de Down y Turner son ms conocidos como cau-
sas de talla baja. Recuerda que, por el contrario, el sndrome de Klinefelter
(XXY), igual que el sndrome XYY, produce talla alta.
P180 MIR 2006-2007
Con respecto al dfcit de hormona de crecimiento en la infancia, los pa-
cientes que aquejan dicho dfcit muestran una velocidad de crecimiento
inferior a la normal y su curva de crecimiento se desva progresivamente
del canal normal. Pero si el dfcit es congnito, se manifesta a partir de
los 6-12 meses de edad, siendo la talla y el peso al nacer normales. Eso es
debido a que en esta etapa de la vida el crecimiento no depende de GH,
sino de la insulina. La mayora responden al tratamiento con GH sinttica
con una aceleracin de la velocidad de crecimiento hasta un lmite normal
o, incluso, por encima de lo normal.
P188 MIR 2005-2006
Pregunta fcil de un tema constante en el MIR: repsala porque no puedes
permitirte fallarla. Se trata, simplemente, de valorar una analtica general
en un paciente peditrico.
nicamente hemos de determinar el tipo de deshidratacin (isotnica,
hipotnica o hipertnica) y el tipo de acidosis (respiratoria, metablica o
mixta). Para contestar a este tipo de preguntas basta con conocer los ni-
veles normales de sodio, pCO
2
y bicarbonato. En general, qudate con las
cifras 35-45, puesto que te sern tiles para varios de estos niveles:
Na: 135-145 mEq/l.
pCO
2
: 35 - 45 mmHg (aprovecho para recordarte los niveles del pota-
sio que son 3,5-5, pero para recordarlos utiliza 3,5-4,5).
El pH normal: 7,35-7,45.
El bicarbonato oscila alrededor de 25 (21-28), pero en el MIR te darn
niveles sufcientemente extremos como para que decidas si est alto,
bajo o normal.
Sabiendo esto veamos los datos que nos ofrecen para responder la pregunta:
El Na es de 137, por tanto, est comprendido entre los valores norma-
les: se trata entonces de una deshidratacin isotnica.
El pH es de 7,20, consecuentemente, el paciente se encuentra en aci-
dosis.
El tipo de acidosis viene determinada por los niveles de pCO
2
y de HCO
3
.
Si el paciente no ventila adecuadamente (un ejemplo clsico del MIR sera
la intoxicacin por sustancias sedantes), la pCO
2
aumentar y hablaremos
de acidosis respiratoria. Si el paciente est perdiendo HCO
3
por alguna
va (el ejemplo tpico sera el de la pregunta, con diarrea y vmitos; pero
tambin existen causas renales como la acidosis tubular proximal o tipo II),
ste disminuir dando lugar a una acidosis conocida como metablica. En
el caso de nuestra pregunta tenemos varios datos para decir que se trata
de una acidosis metablica: por un lado el HCO
3
est bajo (11) y, por otro,
la clnica de diarrea y vmitos. Fjate que la pCO
2
lejos de estar aumentada
como correspondera a una acidosis respiratoria, est descendida (pCO
2

de 25); esto se explica por la compensacin que est realizando el pulmn
eliminando CO
2
del organismo para corregir de esta forma el pH (recuerda
estos binomios: hipoventilacin-acidosis/hiperventilacin-alcalosis).
P188 (MIR 05-06) Tipos de deshidratacin
Prdida Agua = solutos Solutos > agua Agua > solutos
Osmolaridad 285 mosm/l <270 mosm/l >300 mosm/l
Na 130-150 mEq/l <130 mEq/l >150 mEq/l
Mucosas secas +
Mucosas secas Pliegue ++ Mucosas secas ++
Pliegue + Fontanela deprimida + Sed
Clnica Fontanela deprimida Hipotensin + NRL
Hipotensin Oliguria + (hemorragia subdural)
Oliguria Convulsiones
ISOTNICA HIPOTNICA HIPERTNICA
P171 MIR 2003-2004
Aunque sea aparentemente compleja, esta pregunta se puede contestar
fcilmente conociendo la defnicin de talla baja o hipocrecimiento.
Se habla de hipocrecimiento cuando la talla se encuentra por debajo del
percentil 3 para la edad y sexo. La paciente del caso tiene una talla en el
P10, con lo que en principio debemos asumir que su crecimiento es normal
(opcin 1 correcta). Los dems datos del enunciado no hacen sino apoyar
este diagnstico. La paciente es una nia de diez aos que todava no ha
iniciado el desarrollo puberal (estadio 1 de Tanner). Se considera que en
el perodo prepuberal inmediato la velocidad de crecimiento debe ser al
menos de 4 cm/ao (la de la paciente es de 5 y, por consiguiente, normal).
Por otro lado, la talla gentica de la paciente (determinada por la talla me-
dia de los progenitores) est en el P15, bastante prxima a su talla real.
Finalmente, se asume que la edad sea de un paciente es normal cuando
se encuentra en un intervalo de +/ 1 ao en torno a la edad cronolgica.
P176 MIR 2003-2004
Pregunta relativamente fcil si se tienen en cuenta las nociones bsicas de
alimentacin del lactante.
Hemos de fjarnos en el matiz que dan a lactancia materna en la respuesta
2: prolongada.
El RN tiene unos depsitos de hierro sufcientes para los primeros meses de
vida. Esto hace que la lactancia materna sea sufciente para evitar la apari-
cin de anemia ferropnica en ese perodo. Pero cuando stos empiezan a
agotarse, a pesar de la buena absorcin del hierro de la leche materna, ste
debe aportarse de forma extra mediante alimentos enriquecidos (como
frmulas de continuacin).
Tambin es conveniente en esta pregunta recordar que tras los primeros
das de vida, la causa ms comn de anemia es el dfcit de hierro.
504 Desgloses comentados
Pediatra
P197 MIR 2002-2003
Pregunta fcil sobre la alimentacin complementaria del lactante.
Sobre la alimentacin complementaria lo ms importante es que se debe
iniciar a partir del 4-6 mes; no antes porque el lactante todava tiene pro-
blemas de deglucin, supone una sobrecarga de solutos y el aparato di-
gestivo todava no est preparado; pero tampoco despus, porque la leche
sola ya no cubre las necesidades del lactante.
Lo primero que se debe introducir son los cereales sin gluten y se debe
esperar al 6-8 mes para comenzar con los cereales con gluten (por eso las
opciones 3 y 5 son falsas).
En cuanto al resto de opciones, en la 1 ya hemos comentado que precisamen-
te esos cereales son los que no tienen gluten; la opcin 2 no se ha relacionado
con la dermatitis atpica, su introduccin precoz s que se ha relacionado con
formas graves de celaca; la opcin 4 nos habla de la hidrlisis mediada por la
amilasa pancretica que no infuye para nada sobre los cereales.
Por eso fue anulada, ya que todas las opciones son falsas.
P190 MIR 2001-2002
La muerte es un proceso que, obviamente, el nio vive de manera diferente al
adulto. Ante la separacin defnitiva (la muerte de un progenitor, por ejemplo) la
mayora de los nios preadolescentes no parecen sufrir el tpico proceso de duelo.
El duelo del nio puede estar enmascarado por una conducta que no es
tpica de los adultos. Muchos nios continan sus actividades cotidianas y
el principal mecanismo de defensa ante la catstrofe es la negacin, tanto
manifesta como inconsciente, mantenida por el deseo y la esperanza m-
gicos de reaparicin y de reencuentro.
Algunos nios muestran sentimientos hostiles y de ira hacia el progenitor
que sobrevive, y tienden a identifcarse con el progenitor perdido y a idea-
lizarlo, a veces con fantasas de reencuentro que acompaan a la negacin.
Otra posibilidad es que los nios demuestren un dolor considerable al mo-
rir uno de los padres, o tras un lapso de tiempo, una vez que el mecanismo
de defensa de la negacin deja de ser efcaz.
Los nios menores de cinco aos creen que la muerte es reversible,
y posiblemente piensan que los muertos vuelven a la vida o en los fan-
tasmas. En la fase siguiente, hasta los ocho o nueve aos, la muerte se
personifca, como por ejemplo "el Coco" que castiga y se venga. Slo pasa-
da esta edad el nio comprende la muerte de forma realista, como un
proceso biolgico fnal y universal (respuesta 3 correcta).
T3
Aparato respiratorio
P173 MIR 2009-2010
La OMS defne como caso clnico de tosferina a aquel sujeto con tos que
dura al menos dos semanas, con uno o ms de los siguientes sntomas:
Accesos de tos paroxstica.
Estertor al inspirar (gallo).
Vmitos postusivos y sin otra causa obvia.
El paciente de la pregunta cumple con varios criterios: tos seca espasm-
dica de dos semanas de evolucin, que incluso le han provocado hemorra-
gias subconjuntivales por el esfuerzo.
El agente microbiolgico principal es la Bordetella pertussis, si bien tambin
se han aislado Bordetella parapertussis y Bordetella bronchiseptica, aunque
en menor proporcin. La tosferina es una de las enfermedades transmisi-
bles ms contagiosas, siendo los lactantes menores de 6 meses, los adoles-
centes (como el caso de la pregunta) y los adultos los ms susceptibles de
padecer la enfermedad. Estos ltimos actan como transmisores al lactan-
te, donde provoca mayor morbilidad. Normalmente, la tos ferina del adulto
cursa como un cuadro catarral inespecfco.
Pese a que el paciente est correctamente vacunado, se sabe que la inmu-
nogenicidad de la vacuna es relativamente variable. La efcacia frente a la
enfermedad clnica es del 70-90% durante 2-5 aos, y va disminuyendo
con el tiempo, no prolongndose ms all de 12 aos tras la administracin
de la ltima dosis de vacuna. El paciente recibi la ltima dosis de tos ferina
a los 6 aos, con lo cual la inmunidad esperada es menor. Por todo ello,
la respuesta correcta es la opcin 4: un macrlido, como tratamiento ms
efcaz frente al gnero Bordetella.
La opcin 1 (asma) se descarta porque la auscultacin es normal y no hay
signos de difcultad respiratoria. Seran precisos ms estudios complemen-
tarios (opcin 2 falsa) como muestras para aislamiento de Bordetella en
una muestra clnica. Habra que estudiar a los contactos (opcin 3 falsa)
para descartar contagios. Por ltimo, la hemorragia conjuntival (opcin 5)
es un dato tpico. Se debe a la hiperpresin endotorcica, secundaria a los
accesos de tos, que se transmite hacia los vasos conjuntivales.
P190 MIR 2008-2009
Una pregunta laboriosa por la extensin del caso clnico pero, en realidad,
sencilla y referente a un tema caracterstico del Examen MIR. Aunque la
defnicin de bronquiolitis habla de lactantes menores de dos aos con su
primer episodio de difcultad respiratoria, aparece ms en los menores de
seis meses, como sucede en esta pregunta.
Es importante tener en cuenta los sntomas previos para su diagnstico,
porque en los lactantes de ms edad, sobre todo con antecedentes fami-
liares u otros factores de riesgo para hiperreactividad bronquial (dermatitis
atpica, alergias alimentarias, tabaquismo pasivo), la bronquiolitis puede
confundirse con el asma.
Las crisis de broncospasmo, a diferencia de las bronquiolitis, suelen tener
una aparicin brusca; en la auscultacin se escuchan fundamentalmente
sibilancias (sin ruidos de secreciones), y lo ms importante, las crisis se re-
piten con frecuencia. Adems, en este caso nos mencionan un episodio
catarral previo, que tambin respalda la respuesta 2.
P192 MIR 2008-2009
El cuadro que nos describen corresponde a una tos ferina. Esta enfermedad
es una infeccin producida principalmente por Bordetella pertussis. Afecta
preferentemente a menores de un ao. Clnicamente produce un prdro-
mo inespecfco, de tipo catarral, y luego aparece la fase que caracteriza a
la enfermedad, que es la de tos paroxstica. Origina accesos repentinos de
tos, con series repetitivas de mltiples toses enrgicas, acompandose
despus de un ruido inspiratorio que recibe el nombre de gallo, al pasar
aire a travs de una glotis cerrada. La tos, en ocasiones, se asocia con vmi-
505
Desgloses comentados
Pediatra
tos (tos emetizante). Las complicaciones de la tos ferina son:
Neumona: la ms frecuente. Suele ser por sobreinfeccin bacteriana
secundaria.
Convulsiones, cuya causa no est bien establecida.
Otras: prolapso rectal, hemorragia subconjuntival, hernia umbilical,
epistaxis. Estas complicaciones se deben a la intensa presin positiva
intratorcica que se produce en relacin con la tos.
A pesar de tratarse de una infeccin bacteriana, es caracterstica la presencia
de linfocitosis en el hemograma, como podemos ver en este caso. El trata-
miento antibitico de la tos ferina es la eritromicina durante dos semanas.
P227 MIR 2007-2008
Una pregunta ms sencilla de lo que parece, dado que se presta a la deduc-
cin lgica. Nos preguntan por el virus que no causa neumonas (o lo hace
con escasa probabilidad). Entre las opciones que nos exponen, sera lgico
pensar que el virus respiratorio sincitial pueda producir infecciones bron-
copulmonares (de hecho, como sabes, es causa de bronquiolitis). Tambin
sabemos que el virus Infuenzae (gripe) ocasiona cuadros respiratorios, as
que no es de extraar que el parainfuenzae produzca cuadros parecidos.
Siguiendo este razonamiento, la duda estara entre el adenovirus y el rota-
virus. Sobre ellos, deberas saber:
Adenovirus: estos virus producen infecciones respiratorias en ni-
os y reclutas (colectivos cerrados, como campamentos militares,
colegios.) y otros muchos posibles cuadros (diarrea, conjuntivitis,
faringitis).
Rotavirus: es la causa ms comn de diarrea de origen vrico en el lactan-
te, pero es rarsimo que produzca infecciones respiratorias (respuesta 4).
P192 MIR 2006-2007
Con respecto a la fbrosis qustica, el diagnstico se basa en las manifesta-
ciones clnicas tpicas (respiratorias, digestivas y genitourinarias) o antece-
dentes de fbrosis qustica en un hermano o prueba de cribado neonatal
positivo, ms datos de laboratorio de disfuncin de CFTR (test del sudor
positivo en dos resultados positivos en das distintos) o dos mutaciones de
FQ en estudio gentico o alteraciones en la diferencia de potencial nasal.
P192 (MIR 06-07) Diagnstico de fibrosis qustica
P186 MIR 2005-2006
Pregunta de difcultad fcil-media en forma de caso clnico de bronquio-
litis.
La bronquiolitis es una enfermedad viral que cursa con obstruccin infa-
matoria de las pequeas vas areas. Se defne como el primer episodio
de difcultad respiratoria con sibilantes espiratorios que tiene lugar en
un nio < 2 aos (el nio de la pregunta tiene dos meses), con sntomas
de infeccin de va respiratoria de tipo vrica. El virus respiratorio sincitial
(VRS) es el agente causal ms frecuente.
La fuente de infeccin suele ser familiar y la transmisin se produce por va
respiratoria. Los nios mayores y adultos no presentan difcultad respirato-
ria a pesar de la infeccin, dado que toleran mejor el edema bronquiolar.
En efecto, la contribucin de la va area de pequeo calibre a la resistencia
total es mayor en los lactantes, y durante una infeccin por el VRS se pro-
duce una obstruccin bronquiolar causada por edema, acmulo de moco
y detritus celulares, con el resultado fnal de disminucin del radio de la va
area y aumento refejo de la resistencia al paso de aire.
Es frecuente el antecedente de infeccin respiratoria leve unos das an-
tes (hace tres das tuvo mocos, tos, estornudos, etc.) para aparecer pos-
teriormente tos, difcultad respiratoria e irritabilidad. Habitualmente no
existe febre, pero puede haber febrcula. Debido a la taquipnea, el lactan-
te puede manifestar rechazo del alimento. En la exploracin encontramos
signos de difcultad respiratoria como aleteo nasal, tiraje, etc. En la auscul-
tacin se oye una espiracin alargada y sibilancias. La disminucin signi-
fcativa de los ruidos respiratorios indica obstruccin casi completa de las
vas areas y constituye, por tanto, un signo de gravedad.
El diagnstico es habitualmente clnico apoyado en la radiologa. Se asume
que bronquiolitis es todo aquel primer episodio de difcultad respiratoria
baja que sufre un lactante menor de dos aos, por otro lado, sano. La radio-
loga muestra hiperinsufacin pulmonar. Se puede demostrar el virus
en secreciones nasofarngeas por inmunofuorescencia y elevacin de los
ttulos de anticuerpos en sangre o cultivo.
Para el tratamiento es esencial que recuerdes que los broncodilatadores
son de muy dudosa efcacia, puesto que la fsiopatologa de la enfermedad
no es la broncoconstriccin (como ocurre en el asma). Lo que ms les be-
nefcia es la administracin de oxgeno. La adrenalina inhalada puede ser
til al producir vasoconstriccin y, por consiguiente, ayudar a disminuir el
edema. La ribavirina inhalada se reserva para casos graves. En resumen,
recuerda que no estn indicados ni broncodilatadores, ni corticoides,
ni antibiticos ni sedantes.
P186 (MIR 05-06) Diagnstico diferencial entre bronquiolitis y asma
506 Desgloses comentados
Pediatra
El principal diagnstico diferencial de la bronquiolitis se debe realizar con
el asma. El asma:
Es raro en menores de un ao.
No hay infeccin previa.
Suele existir historia familiar.
Las crisis son repetidas (solamente se llama bronquiolitis al primer
episodio de difcultad respiratoria de un lactante menor de dos aos).
El comienzo es brusco.
P190 MIR 2002-2003
Pregunta de difcultad media sobre la patologa congnita del aparato res-
piratorio. No es un tema muy preguntado en el MIR, pero donde las carac-
tersticas de los distintos tipos de cuadros son muy parecidas.
La laringomalacia y la traqueomalacia son las causas ms habituales de estri-
dor congnito. El diagnstico diferencial entre ambas se realiza por el tipo de
estridor; as, un estridor inspiratorio es ms tpico de una afectacin de vas
altas (larngeo), mientras que si es tanto inspiratorio como espiratorio es tpico
de afectacin de vas intermedias (trquea). La afectacin de vas bajas (bron-
quios, bronquiolos) lo que suele producir son sibilancias espiratorias.
P190 (MIR 02-03) Patologa larngea en el nio
El diagnstico se establece por laringoscopia directa. En general, el proce-
so se resuelve espontneamente y no precisa tratamiento especfco. Si hay
difcultades para la alimentacin, puede ser necesaria la alimentacin por
sonda. Muy excepcionalmente se utiliza la traqueotoma.
En consecuencia, en este caso la opcin correcta sera el tratamiento
conservador, puesto que es el primer paso a realizar. Las opciones 1 y 4
son maniobras invasivas que slo se utilizaran en casos ms graves; la
opcin 2 hace referencia al tratamiento quirrgico del refujo gastroeso-
fgico.
P195 MIR 2002-2003
Pregunta fcil sobre la fbrosis qustica.
Se trata de una pregunta directa sobre un diagnstico ya dado, la fbrosis
qustica, y donde nos la describen clnicamente como enfermedad mul-
tisistmica caracterizada por obstruccin crnica de las vas respiratorias,
siendo la principal causa de enfermedad pulmonar crnica grave en la
infancia y tambin la responsable de la mayor parte de las insufciencias
pancreticas exocrinas en las primeras etapas de la vida.
Desde el punto de vista gentico, es la enfermedad hereditaria ms fre-
cuente y letal de la raza blanca. Se hereda de forma autosmica recesiva,
estando el gen localizado en el brazo largo del cromosoma 7 (una regla
para recordarlo es: FQ, si damos la vuelta a la F nos queda 7q). Por eso, la
opcin incorrecta es la 3.
El resto de opciones nos describen las caractersticas patognicas de la en-
fermedad.
Desde el punto de vista clnico, lo ms preguntado suele ser la frecuencia
de sobreinfecciones bacterianas; teniendo en cuenta que el germen ms
frecuente: P. aeruginosa, el ms precoz: S. aureus y el ms grave: B. cepacea.
Asimismo, la insufciencia respiratoria crnica lleva al desarrollo en estos
pacientes de un cor pulmonale.
Desde el punto de vista diagnstico se requieren dos criterios:
Manifestaciones clnicas tpicas o antecedentes de FQ en un hermano
o pruebas de cribado neonatal positivas, ms algn dato de laborato-
rio de disfuncin de CFTR.
Test de sudor positivo; dos mutaciones conocidas de FQ en estudio
gentico; alteracin en la diferencia de potencial nasal. Repasa la fgu-
ra de la pregunta 192, MIR 06-07.
El tratamiento es fundamentalmente sintomtico.
T4
Aparato digestivo
P135 MIR 2011-2012
Pregunta de difcultad media debido a ser un tema que haca aos no apa-
reca en el MIR, si bien con los contenidos del manual, abordable. Fjate que
te presentan un neonato con imposibilidad al paso de sonda nasogstrica
en el paritorio. La sospecha de atresia esofgica se confrma si sigues le-
yendo el enunciado donde menciona bolsn esofgico atrsico. Para fliar
ante qu tipo de atresia nos encontramos (fgura), nos comentan que la
neumatizacin intestinal es NORMAL. Con estos datos excluimos las fstu-
las tipo I: donde slo existe atresia y al no haber fstula no puede haber
paso del aire al intestino; tipo II: la fstula proximal permite el paso nica-
mente a va respiratoria sin neumatizacin intestinal; y tipo V: donde no
507
Desgloses comentados
Pediatra
existe bolsn esofgico atrsico. Nos quedara la tipo III y la IV, la primera
con una frecuencia del 87% y la tipo IV con frecuencia inferior al 1%. De
todas maneras esta pregunta no requiere un conocimiento tan profundo,
ya que la tipo III es la ms frecuente con diferencia (87%), la que has de
saber, ya que es la que con probabilidad te preguntaran (recuerda la regla:
la aTRESia esofgica ms frecuente es la tipo TRES).
P135 (MIR 11-12) Atresia de esfago
La prioridad de la ciruga es evitar la neumopata causada por las mi-
croaspiraciones de cido gstrico por la fstula distal, cada vez que el
neonato inspira ejerciendo presin negativa (opcin 3 correcta). De he-
cho, hasta que el neonato est estable y se opere, se coloca en decbito
prono con objeto de evitarlas. Fjate que las opciones 1 (imposibilidad
de deglutir saliva) y 4 (imposibilidad de alimentacin enteral) vienen a
ser lo mismo y, por ello, son descartables, adems de que no suponen
una prioridad quirrgica. La opcin 2 (malformaciones cardacas fre-
cuentemente asociadas) se excluye con el enunciado tras evaluacin
diagnstica que descarta otras anomalas.... En la clnica, hay que tener
presente la asociacin VACTERL (malformaciones Vertebrales, Anorrec-
tales, Cardacas, Traqueales y Esofgicas, Renales y radiales -Limb en in-
gls-) presente en un 30-50% de las atresias esofgicas; de ah que lo
mencionen en el enunciado.
Por ltimo, recuerda que una de las complicaciones postoperatorias es
el reflujo gastroesofgico (MIR 05-06, 191), por anomalas intrnsecas
de la funcin esofgica. Otras como traqueomalacia, estenosis esof-
gica, fstula de la anastomosis o de la fstula traqueoesofgica son ms
infrecuentes.
P136 MIR 2011-2012
En el caso que nos comentan, nos falta informacin tanto de la anam-
nesis, como de la exploracin fsica, por lo que la pregunta fue anulada
por el Ministerio. Ante un nio con estreimiento desde el nacimien-
to, debemos diferenciar dos causas principalmente: un estreimiento
funcional y una enfermedad de Hirschsprung. Si estuviramos ante una
enfermedad de Hirschsprung describiran a un paciente que present
un retraso en la evacuacin del meconio, que suele precisar estimu-
laciones para realizar deposicin, que probablemente presentara un
retraso ponderal y que en el tacto rectal se apreciara la ampolla va-
ca y una hipertona del esfnter anal. Sin embargo, nos encontramos
ante un lactante que realiza deposiciones de consistencia blanda de
forma espontnea sin requerir estimulacin ni laxantes, por lo que pro-
bablemente se trate de un estreimiento funcional. El hecho de que
las realice cada 5-6 das puede considerarse un ritmo intestinal normal.
Por tanto, las opciones que deberamos valorar son la 4 y la 5, aunque
nos faltaran datos para contestar una de las dos con certeza. A conti-
nuacin, se exponen las principales diferencias entre el estreimiento
funcional y la enfermedad de Hirschsprung.
P136 (MIR 11-12) Diferencias entre estreimiento funcional
y enfermedad de Hirschsprung
ESTREIMIENTO
FUNCIONAL
ENFERMEDAD
DE HIRSCHSPRUNG
Inicio al nacimiento Raro Frecuente
Enterocolitis No Posible
Tamao de las heces Grandes Acintadas o normales
Retraso ponderal Raro Frecuente
Ampolla rectal Llena Vaca
Incontinencia fecal Frecuente Rara
Tono del esfnter Variable Elevado
P153 MIR 2010-2011
Pregunta fcil sobre un tema tpico, repetido en varias convocatorias. Este
tipo de preguntas no se pueden fallar.
El cuadro clnico corresponde a una estenosis hipertrfca del ploro: lac-
tante varn que debuta en torno a los 20 das de vida con vmitos alimen-
tarios (no biliosos), a chorro o proyectivos, que le provocan una alcalosis
metablica hipoclormica (se pierde HCl), con tendencia a la hipopotase-
mia, un estado permanentemente hambriento e irritable, deshidratacin
y desnutricin.
En la exploracin fsica, se palpa en algunos casos la oliva pilrica. La
tcnica de eleccin para confirmar el diagnstico es la ecografa abdo-
minal (respuesta 3 correcta). Cuidado con la respuesta 1, la Rx simple de
abdomen es til en el diagnstico diferencial, pudiendo ver una nica
burbuja tanto en la hipertrofia como en la atresia pilrica y una doble
burbuja en la atresia duodenal, pero nos piden la prueba complemen-
taria de eleccin.
El tratamiento preoperatorio consiste en fuidoterapia i.v. para corregir las
alteraciones hidroelectrolticas, siendo el tratamiento curativo la piloro-
miotoma de Ramsted.
Hay que tener presente en la patologa digestiva peditrica los diag-
nsticos diferenciales, en este caso con la atresia pilrica (vmitos no
biliosos desde las primeras tomas), la atresia duodenal (vmitos biliosos
precoces, regla de la D: duodenal, doble burbuja, sndrome de Down), el
reflujo gastroesofgico (vmito atnico de contenido alimentario desde
la primera semana de vida, la pHmetra, respuesta 2, es la prueba ms
508 Desgloses comentados
Pediatra
sensible y especfica pero no la primera a realizar) y con la insuficiencia
suprarrenal congnita (vmitos desde el nacimiento con alcalosis meta-
blica hiperpotasmica).
P153 (MIR 10-11) Diagnstico diferencial de la patologa digestiva
peditrica
PATOLOGA TIPO DE VMITO RADIOGRAFA
Estenosis hipertrfca
de ploro
NO bilioso
Distensin gstrica
con escaso gas distal
Atresia de ploro NO bilioso Imagen de NICA burbuja
Atresia de duodeno BILIOSO Imagen de DOBLE burbuja
P170 MIR 2009-2010
Un tema que, desde hace muchas convocatorias, no apareca en el examen
MIR (en concreto desde 1996). No obstante, la pregunta es bastante senci-
lla, ya que la clnica del paciente tiene poco que ver con la que producira
una fsura anal (respuesta 1 falsa).
Defnimos intolerancia alimentaria como aquella reaccin adversa que
aparece tras la ingesta de alimentos, que no est mediada por IgE. Los sn-
tomas son exclusivamente digestivos: diarrea, rectorragia, vmitos, dolor
abdominal, etc.
La alergia alimentaria, a diferencia de la intolerancia, s est mediada por
IgE. Clnicamente, aparte de los sntomas digestivos, podremos verlos de
tipo alrgico: urticaria, angioedema, broncospasmo e incluso anaflaxia, en
los casos ms graves.
La leche de vaca es el alimento ms frecuentemente implicado en los pro-
cesos de intolerancia y alergia alimentaria. La protena ms relacionada
con estos fenmenos es la beta-lactoglobulina, seguida de la alfa-lactoal-
bmina y la casena.
El diagnstico es fundamentalmente clnico. Los sntomas aparecen cuando
el nio toma leche de vaca y desaparecen cuando este alimento es excluido
de la dieta. El pronstico de esta entidad es favorable, ya que desaparece a lo
largo de los primeros aos de la vida. Observa que, en el nio que nos plan-
tean, solamente existen sntomas digestivos, por lo que se tratara de una
intolerancia. Por lo tanto, no estara mediada por IgE, como dice la respuesta
3. De hecho, la intolerancia es ms frecuente que la alergia.
El tratamiento consiste en sustituir la frmula normal por un hidrolizado de
protenas lcteas. Los sntomas alrgicos pueden precisar un manejo espe-
cfco, como antihistamnicos e incluso adrenalina subcutnea, si hubiese
angioedema. En casos seleccionados se ha planteado el uso de cromogli-
cato sdico.
P175 MIR 2009-2010
La enfermedad de Hirschprung o aganglionosis colnica es todo un clsico
en las preguntas del MIR de Pediatra. Antecedentes tpicos son estrei-
miento desde el perodo neonatal (tapn meconial) y a lo largo del primer
ao de vida (esta paciente tiene 4 meses), debido a la peristalsis inadecua-
da del segmento aganglinico. Otros signos tpicos seran la distensin ab-
dominal (megacolon) debido a la retencin fecal y ausencia de gas distal,
donde generalmente asienta el tramo aganglinico. Entre las complicacio-
nes probables nos encontramos la de la pregunta: obstruccin. Inicialmen-
te se toman medidas de desobstruccin, hasta la realizacin de pruebas
ms especfcas que confrmen la sospecha diagnstica (enema, manome-
tra y biopsia): opcin 3.
P180 MIR 2008-2009
Caso clnico tpico sobre una estenosis hipertrfca del ploro.
El sntoma cardinal en esta enfermedad lo constituyen los vmitos. stos
sern no biliosos, proyectivos, y aparecen despus (o casi despus) de las
tomas. Suelen comenzar a partir de los 20 das de vida. Debido a la prdi-
da de hidrogeniones y cloruros que supone el vmito, tiende a producirse
una alcalosis metablica hipoclormica. En este caso clnico, las pequeas
estras sanguinolentas pueden ser debidas a la intensidad del vmito, que
puede producir hiperpresin a nivel de los vasos de la mucosa esofgica,
dandose algunos y produciendo un leve sangrado.
En la exploracin podemos encontrar grados variables de deshidratacin
o desnutricin, as como ictericia o subictericia. En algunos casos, incluso
es palpable la oliva pilrica a nivel epigstrico, bajo el reborde heptico.
La tcnica de eleccin para confrmar el diagnstico es la ecografa abdo-
minal (respuesta 4 correcta). Se detectar un grosor del msculo pilrico
por encima de lo normal (> 4 mm), as como un aumento de la longitud
global del canal pilrico.
P182 MIR 2008-2009
Una pregunta difcil sobre el diagnstico de la enfermedad celaca.
Ante la sospecha de esta enfermedad, lo habitual es realizar una determi-
nacin de anticuerpos (recuerda que los ms sensibles y especfcos son
los IgA-antitransglutaminasa). Si esta determinacin resulta positiva, esta-
ra indicado realizar una biopsia intestinal, que tambin podra plantearse
ante una determinacin negativa, pero con una frme sospecha clnica. Por
tanto, el papel de los anticuerpos en el diagnstico de esta enfermedad
tiene valor fundamentalmente como cribado.
La biopsia intestinal es una prueba necesaria para establecer el diagns-
tico. Se realizara por lo menos en una ocasin, estando el paciente con-
sumiendo gluten. En este caso clnico ya tenemos una biopsia intestinal
compatible con enfermedad celaca (la duda estara en la necesidad de
realizarla por segunda vez). La confrmacin de la recuperacin histolgica
despus de la retirada del gluten (segunda biopsia) se realizara en casos
dudosos. No obstante, en este caso lo tenemos bastante claro: buena evo-
lucin clnica despus de retirar el gluten de la dieta y negativizacin de los
anticuerpos, por lo que en principio bastara continuar con la dieta.
P186 MIR 2007-2008
Pregunta con un caso clnico tpico de refujo gastroesofgico madurativo
o fsiolgico.
La manifestacin clnica ms usual es la regurgitacin en la mayora de las
tomas a las pocas semanas de vida (la mayora empiezan ya en la primera
semana). Dado que el paciente presenta un adecuado desarrollo pondoes-
tatural, hablamos de un refujo gastroesofgico fsiolgico debido a un es-
fnter esofgico inferior incompetente por inmadurez (opcin 4 correcta).
509
Desgloses comentados
Pediatra
La estenosis pilrica es un cuadro mucho ms agudo, con vmitos proyec-
tivos no biliosos, que comienza en torno a las tres semanas de vida con
riesgo de deshidratacin para el neonato (opcin 2 falsa). Tanto la opcin
1 como la 3 hacen referencia a reacciones adversas frente a la protena de
la leche de vaca, y por ello no guardaran tan buena evolucin: se afecta-
ran el peso y la talla. En la intolerancia, es comn el hallazgo de heces con
sangre. La alergia puede provocar urticaria, angioedema, broncoespasmo,
e incluso anaflaxia como forma grave.
Por ltimo, la opcin 5, el vlvulo intestinal, se presentara como un cuadro
obstructivo y clnica de peritonismo. No permitira una evolucin tan sola-
pada en el tiempo (hablan de dos meses y medio), sino mucho ms aguda.
P188 MIR 2007-2008
La intolerancia a disacridos provoca que stos no se absorban adecuada-
mente. Mediante un efecto osmtico, atraen agua hacia la luz intestinal, de
manera que las heces se vuelven acuosas (diarrea postprandial, opcin 1 co-
rrecta). A su vez, al ser fermentados por la fora saprofta del colon, las heces
se vuelven cidas (por lo que aparece secundariamente eritema perianal) y
se acompaan de meteorismo, debido al gas generado en la fermentacin;
as pues, stas se eliminan explosivamente. En esto se fundamenta la prueba
diagnstica del H
2
espirado (opciones 2, 3 y 4 correctas).
Las heces esteatorreicas (grisceas, brillantes, adherentes, fotantes) son
propias de los cuadros clnicos en los que existe malabsorcin de lpidos
(opcin 5 falsa). En Pediatra, no olvides nunca la celiaqua ni la fbrosis
qustica.
P183 MIR 2005-2006
Pregunta de difcultad fcil-media acerca del diagnstico de laboratorio de
la intolerancia-malabsorcin de lactosa.
La malabsorcin de lactosa se debe a un dfcit de lactasa, que puede ser
congnito, pero que lo ms comn es que sea secundario a procesos que
lesionan de forma difusa el epitelio intestinal (infecciones, enfermedad ce-
laca...). Los disacridos no hidrolizados se acumulan en el intestino, donde
las bacterias producen cidos orgnicos e hidrgeno, lo que atrae agua a
la luz intestinal y se genera diarrea osmtica.
Clnicamente se manifesta por heces espumosas, con pH bajo, ricas en
azcar y que suelen excoriar las nalgas. Se produce tambin distensin ab-
dominal, borborigmos y ocasionalmente dolor clico. Para el diagnstico
podemos usar:
Deteccin de cuerpos reductores en heces mediante la prueba del
clinitest que se cuantifca segn cambio de color. Recuerda que la
sacarosa no es un azcar reductor y, por tanto, si se sospecha into-
lerancia a la sacarosa, previamente debe hidrolizarse con cido clor-
hdrico.
Determinacin de pH fecal inferior a 5,6 o determinacin de cido lc-
tico en heces.
Actividad de disacaridasas medida en la biopsia intestinal disminuida.
Test de H
2
en aire espirado tras sobrecarga oral de lactosa.
El test de la ureasa en la biopsia intestinal se usa en el diagnstico de
infeccin por H. pylori. Se trata de un mtodo invasivo, puesto que se hace
a partir de una biopsia de antro y cuerpo gstrico. Se basa en que la ureasa
que origina la bacteria hidroliza la urea y cambia el color de un indicador,
lo que signifca que hay infeccin por H. pylori. Por consiguiente, nada tiene
que ver con el dfcit de lactasa.
P191 MIR 2005-2006
Se trata de una pregunta de elevada difcultad sobre un tema poco pre-
guntado en el MIR.
P191 (MIR 05-06) Atresia esofgica
La atresia y fstula traqueoesofgica ocurren con una incidencia 1/3.000-
4.500 RN vivos. Existen diversas clasifcaciones, aunque la ms aceptada es
la clasifcacin de Ladd, que distingue cinco tipos:
Tipo 1: atresia sin fstula.
Tipo 2: fstula proximal y atresia distal.
Tipo 3: atresia proximal y fstula distal.
Tipo 4: doble fstula.
Tipo 5: fstula sin atresia.
La forma ms frecuente es la III, con casi el 85% de los casos.
Debemos sospechar esta patologa ante:
Existencia de polihidramnios.
Imposibilidad para pasar sonda nasogstrica en el paritorio.
Salivacin excesiva.
Cianosis y atragantamiento con las tomas.
Si existe una fstula traqueoesofgica distal, aparecer una distensin ab-
dominal importante, mientras que si no existe fstula distal, el abdomen
estar excavado. Las formas con fstula proximal cursan con aspiraciones
masivas con la alimentacin. La fstula sin atresia (en H) puede cursar de
forma ms larvada y manifestarse como neumonas recurrentes.
510 Desgloses comentados
Pediatra
El tratamiento ha de ser quirrgico. Durante el preoperatorio: posicin en
decbito prono, aspiracin continua del bolsn esofgico y medidas gene-
rales. La correccin quirrgica suele realizarse en dos tiempos.
Tras la ciruga la complicacin ms frecuente ser los trastornos de la mo-
tilidad esofgica y, en particular, el refujo gastroesofgico. Con menos fre-
cuencia aparecen traqueomalacia (respuesta 5), estenosis (respuesta 2) o
refstulizacin (respuestas 1 y 3).
P192 MIR 2005-2006
Se trata de una pregunta clsica de Pediatra que no podemos fallar el da
del examen.
Nos piden el diagnstico de presuncin y nos dan todos los datos tpicos
de una invaginacin intestinal, as que aprovecha esta pregunta para repa-
sar bien este cuadro.
La edad es tpica, ya que la invaginacin es la causa ms frecuente de
obstruccin intestinal entre los tres meses y los seis aos, sobre todo
en menores de dos aos, siendo raro en perodo neonatal donde la
causa ms habitual de obstruccin intestinal es la enfermedad de Hirs-
chprung.
No olvides que ante una invaginacin intestinal en mayores de seis aos se
han de descartar causas orgnicas como el linfoma intestinal.
La clnica descrita tambin es caracterstica: aparicin brusca de dolor ab-
dominal intenso tipo clico, crisis de llanto, encogimiento de miembros
inferiores y palidez cutnea. A medida que avanza el proceso, el nio se
encuentra ms dbil, somnoliento y aletargado.
En fases iniciales suelen aparecer vmitos. La deposicin de color rojo
oscuro es tambin un signo muy tpico de la invaginacin intestinal en
las primeras horas de evolucin y se conocen como "heces en jalea de
grosella".
En fases avanzadas puede cesar la evacuacin de gas y heces y aparecer
un estado parecido al shock. Recuerda de cara al MIR que la mejor forma
de hacer el diagnstico es la ecografa abdominal, que muestra la ima-
gen en rosquilla, donut o diana en un corte transversal. Otras formas
de hacer el diagnstico son la Rx simple de abdomen o el enema opaco
con el que observamos el signo del muelle enrollado.
Una vez diagnosticada la invaginacin debe procederse a su reduccin
hidrosttica siempre que se cumplan dos condiciones: la invaginacin no
debe ser prolongada y no debe haber datos de perforacin intestinal. Se
podr realizar de dos formas:
Reduccin con aire o suero guiada con ecografa: esta opcin asocia
menor riesgo de complicaciones (perforacin).
Reduccin con enema de bario guiada con control radiolgico: esta
opcin presenta diez veces ms de riesgo de perforacin.
Si existen signos de perforacin, shock, neumatosis intestinal o disten-
sin abdominal de ms de 48 h de evolucin, es preferible la correccin
quirrgica.
Por ltimo, recuerda que la forma ms frecuente de invaginacin intestinal
es la ileoclica.
P192 (MIR 05-06) Invaginacin intestinal. Imagen anatmica
P180 MIR 2004-2005
El cuadro que nos comentan sugiere una hernia de Bochdalek. Este tipo de
hernia diafragmtica es el ms corriente, especialmente en el lado izquier-
do. A travs de la hernia, se produce un paso de rganos abdominales a la
cavidad torcica, que es lo que justifca las imgenes areas que nos descri-
ben en el hemitrax izquierdo (Rx tpica) (respuesta 4 correcta).
Quiz la opcin ms difcil de distinguir sera la malformacin adenoidea
qustica, que genera una imagen parecida (pulmn en queso de Gruyre).
Pero, en este caso, las imgenes areas seran bilaterales.
P180 (MIR 04-05) Hernias diafragmticas
511
Desgloses comentados
Pediatra
P181 MIR 2004-2005
Ten mucho cuidado con este tipo de preguntas. Es bastante frecuente
que intenten confundirte entre el estreimiento funcional y la enferme-
dad de Hirschprung. Recuerda que en el estreimiento funcional s que
es bastante habitual la presencia de encopresis. Sin embargo, sera muy
raro encontrarla en la enfermedad de Hirschprung (respuesta 2 falsa).
La rectorragia leve puede estar justifcada por el traumatismo de la mucosa
rectal en relacin con fenmenos como impactacin y/o heces muy endu-
recidas. El resto de los datos que nos ofrecen s aparecen en el estreimien-
to funcional (ver tabla siguiente).
P181 (MIR 04-05) Diagnstico diferencial del estreimiento
P185 MIR 2004-2005
La enfermedad celaca tiene una base inmunopatolgica (respuesta 1
verdadera), habitualmente asintomtica durante un tiempo, con un pe-
rodo libre de enfermedad que puede durar aos, hasta que aparecen
las primeras manifestaciones clnicas (respuesta 2 cierta). Dentro del
diagnstico diferencial, entraran otras enfermedades digestivas, como
la giardiasis (respuesta 4 cierta), que afecta tambin a duodeno e leon
proximal, compartiendo sntomas como flatulencia, esteatorrea, moles-
tias epigstricas inespecficas, etc.
El diagnstico de enfermedad celaca requiere una biopsia de la
unin duodeno-yeyunal. La lesin histolgica no es especfica, ya
que puede verse en otras enfermedades, pero es muy sugestiva:
atrofia de vellosidades e hiperplasia de las criptas (respuesta 5 co-
rrecta). No obstante, existen anticuerpos que tambin nos dan una
orientacin diagnstica. Los ms especficos son los antitransgluta-
minasa tisular (respuesta 3 falsa).
Repasa con los esquemas siguientes la patogenia y diagnstico de la en-
fermedad celaca.
P185 (MIR 04-05) Patogenia de la enfermedad celaca
P185 (MIR 04-05) Diagnstico de la enfermedad celaca
P174 MIR 2003-2004
Es una pregunta fcil sabiendo la clnica tpica de la estenosis hipertrfca
de ploro EHP.
Nos muestran un neonato de 21 das que presenta vmitos postpran-
diales cada vez ms frecuentes. En un nio de esta edad y con esta
clnica nos debemos plantear el diagnstico de EHP. No olvides que
el vmito no es bilioso y que el nio queda hambriento e irritable tras
el mismo. De entrada podemos descartar la respuesta 4, puesto que
la EHP es de etiologa desconocida, pese a que se ha relacionado con
factores que predisponen a ello como la administracin de prostaglan-
dina E para mantener la permeabilidad del ductus. Si nos fjamos en la
opcin 2, los vmitos persistentes efectivamente generan alteraciones
electrolticas, pero no acidosis metablica, sino alcalosis metablica
hipoclormica.
Si el problema de la EHP es una hiperplasia e hipertrofa de la muscu-
latura lisa del antro gstrico y del duodeno, es lgico pensar que el tra-
tamiento ser quirrgico: pilorotoma de Ramsted (respuesta 3 falsa).
Quedan an las opciones 1 y 5. Sabiendo que la EHP es ms habitual
en primognitos varones de raza blanca en la tercera semana de vida,
podemos afrmar que la 1 es cierta. Recuerda que el polihidramnios no
se relaciona con la EHP.
512 Desgloses comentados
Pediatra
P174 (MIR 03-04) Estenosis hipertrfica de ploro. Imagen anatmica
P175 MIR 2003-2004
Nos enfrentamos a una pregunta accesible acerca de la enfermedad de
Hirschprung, que podremos contestar fcilmente con conocimientos ge-
nerales de su clnica tpica.
Como sabemos, es una enfermedad congnita (respuesta 1 verdadera)
cuyo origen est en una inervacin anmala del intestino, con ausencia de
clulas ganglionares del plexo intramural (respuesta 3 verdadera) por una
migracin anmala de neuroblastos.
P175 (MIR 03-04) Enfermedad de Hirschprung
Supone la causa ms usual de obstruccin intestinal baja en el RN
(1/5.000 RN vivos), con mayor incidencia en varones (respuesta 2 verda-
dera), y puede aparecer asociado a otras alteraciones como sndrome de
Down, Lawrence Moon Bield, Waardenburg y defectos cardiovasculares.
La clnica ms tpica en el perodo neonatal es el retraso en la eliminacin
del meconio. En su diagnstico se emplea la radiografa simple, que mues-
tra distensin de asas con ausencia de aire rectal, el enema opaco, en el
que se aprecia un cambio brusco en el dimetro del colon entre el segmen-
to afecto (estenosado) y porcin sana (distendido) (respuesta 5 verdadera);
con la manometra anorrectal se observa una contraccin del esfnter anal
interno ante el aumento de presin a ese nivel (respuesta 4 falsa), siendo
lo normal una relajacin de dicho esfnter. El diagnstico defnitivo lo da
la biopsia.
P189 MIR 2002-2003
Pregunta fcil sobre un tema tpico en el MIR: la estenosis hipertrfca del
ploro.
Se trata de una pregunta directa sobre las caractersticas de esta enfermedad:
Epdemiologa: es ms frecuente en varones (opcin 1 correcta), so-
bre todo en primognitos.
Clnica: el sntoma principal son los vmitos, no biliosos (opcin 3 in-
correcta), proyectivos (opcin 4 correcta), que comienzan a partir de
los 20 das de vida (opcin 2 correcta). Tras vomitar, el nio queda irri-
table y hambriento. Tambin son tpicas la alcalosis metablica hipo-
clormica (debida al vmito) y la ictericia (opcin 5 correcta) (debida
a la deshidratacin).
Exploracin fsica: palpacin de la oliva pilrica y ondas peristlticas
gstricas.
Diagnstico: clnica, pH, iones (sufciente en el 60-80%), ecografa ab-
dominal (de eleccin), Rx con bario (signo de la cuerda) y Rx simple
(nica burbuja).
Diagnstico diferencial: atresia pilrica (vmitos no biliosos desde las
primeras tomas con imagen de nica burbuja en la radiografa), atresia
duodenal (vmitos biliosos precoces e imagen de doble burbuja en
la Rx, regla de la D: duodenal, doble burbuja, sndrome de Down) y
refujo gastroesofgico (vmito atnico de contenido alimentario ya
en la primera semana de vida).
Tratamiento: preoperatorio: fuidoterapia i.v. para corregir las alte-
raciones hidroelectrolticas y quirrgico (piloromiotoma de Rams-
ted).
P196 MIR 2002-2003
Pregunta bastante sencilla sobre la enfermedad celaca, un tema nada in-
frecuente en el MIR.
Si bien la pregunta es en forma de caso clnico, es todo muy tpico y clara-
mente orientador hacia el diagnstico de enfermedad celaca. La opcin
falsa es sin duda la 5: el tratamiento de la enfermedad celaca es la retirada
total y defnitiva (de por vida) del gluten de la dieta, no concibindose su
retirada gradual. El resto de opciones nos recuerdan datos relevantes so-
bre esta enfermedad: necesidad de, al menos, una biopsia (clsicamente
3); la predisposicin gentica (agrupacin familiar, asociacin con ciertos
HLA) y su heterogeneidad clnica (tanto en tiempo como formas de pre-
sentacin).
Repasa la clnica de la enfermedad celaca con el dibujo siguiente.
513
Desgloses comentados
Pediatra
P196 (MIR 02-03) Enfermedad celaca
P200 MIR 2002-2003
Es una pregunta que hace referencia a las diarreas crnicas y que tiene una
difcultad media.
Es un caso clnico donde se nos presenta a una nia que tras un cua-
dro de gastroenteritis mantiene un proceso diarreico crnico (por
durar ms de dos semanas), sin productos patolgicos, con irrita-
cin del rea perianal (que nos indica un pH bajo). Ante este cua-
dro hemos de sospechar un dficit de disacaridasas secundario a un
proceso que ha lesionado de forma difusa el epitelio intestinal, sien-
do ste el origen ms frecuente. Estos pacientes tambin manifies-
tan con frecuencia dolor abdominal, borborigmos y, ocasionalmente,
dolor clico.
Para confrmar el cuadro podemos realizar un clinitest, test de hidr-
geno en aire espirado o medicin de pH. El tratamiento estriba en la
reduccin de la ingesta de los azcares responsables de la intoleran-
cia a niveles que sean soportables, en este caso de forma transitoria.
Por eso les diremos a los padres que alimenten a su hija con frmulas
sin lactosa durante varias semanas para que se regenere el epitelio
intestinal.
Este proceso no requiere tratamiento antibitico, de hecho podra em-
peorar el cuadro por agresin al epitelio daado, tampoco requiere re-
hidratacin oral, puesto que en ningn caso se nos menciona que pre-
sente prdida de lquidos que justifquen el tratamiento. Los hidrolizados
de protenas seran el tratamiento de eleccin si fuera un paciente que
presentara malabsorcin proteica, con prdida de peso, diarrea, hipopro-
teinemia y edemas.
Si la paciente mostrara un aumento del nmero de las deposiciones con
moco, fbras vegetales sin digerir y granos de almidn, pero sin sangre, leu-
cocitos ni eosinflos, donde no hay perdida de peso, se tratara de una
diarrea crnica inespecfca que afrontaremos tranquilizando a la familia,
pues es un cuadro benigno.
P201 MIR 2002-2003
Es una pregunta bastante sencilla, de manera que no se puede dudar al
contestarla. Forma parte de uno de los temas ms importantes de la pedia-
tra, el aparato digestivo.
La aparicin de dolor abdominal brusco intenso de tipo clico, crisis de
llanto, encogimiento de miembros inferiores y palidez cutnea, es tpica
de la invaginacin intestinal, opcin 4 correcta.
La invaginacin es la causa ms frecuente de obstruccin intestinal en-
tre los tres meses y los seis aos, y la localizacin ms comn es a nivel
ileoclica. Como aparece en este caso, es tpico la presencia de heces
con sangre roja fresca y moco "heces en jalea de grosella". En la explo-
racin clnica aparece como una masa dolorosa, localizada en hipocon-
drio derecho. Para confirmar el diagnstico realizaremos un enema o
una ecografa, que es la prueba de eleccin, donde veremos la imagen
en rosquilla o diana. Respecto al tratamiento, mencionar que se puede
intentar reduccin hidrosttica, o si aparecen signos de perforacin in-
testinal o recidiva se manejar quirrgicamente.
En el supuesto de que fuera una estenosis pilrica, la paciente presentara
vmitos no biliosos a las dos o tres semanas de vida, sin heces con produc-
tos patolgicos.
El resto de las patologas que aparecen se manifestan de diferente manera,
de modo que no plantean dudas de diagnstico diferencial, ya que, por
ejemplo, en la apendicitis aparecera un dolor periumbilical sin vmitos ni
letargia, adems de no ser frecuente a esta edad.
P180 MIR 2001-2002
El refujo gastroesofgico (RGE) es un cuadro relativamente corriente en
el lactante, debido a diversos factores favorecedores: porcin infradiafrag-
mtica del esfago ms corta, estmago ligeramente ms horizontalizado,
menor capacidad gstrica, postura en decbito..., pero no es fsiolgico
(respuestas 1 y 4 incorrectas).
El sntoma ms frecuente es el vmito, que en la mayora de los casos
aparece en la primera semana de vida (recuerda que los vmitos de la es-
tenosis hipertrfca de ploro suelen comenzar hacia la 3 semana). Suele
desaparecer hacia los dos aos de vida.
El RGE puede presentarse tambin como alguna complicacin:
Respiratoria: neumona por aspiracin, tos, sibilancias, etc.
Digestiva: esofagitis con hemorragia digestiva, anemia ferropnica,
dolor, disfagia, etc.
Retraso ponderoestatural.
Sndrome de Sandifer: opisttonos y posturas anmalas de la cabeza,
en relacin con el RGE, bien como respuesta al dolor o como mecanis-
mo de proteccin de la va area (respuesta 2 correcta).
El diagnstico suele realizarse por la clnica y por la respuesta al tratamien-
to. Slo en las formas ms graves o en ausencia de respuesta al tratamiento
emprico se realizan otras pruebas diagnsticas, tales como esofagoscopia
514 Desgloses comentados
Pediatra
y biopsia, esofagograma con bario y pHmetra de 24 horas, siendo esta l-
tima el mtodo ms sensible (respuesta 3 incorrecta).
El tratamiento del RGE se basa en tres pilares:
Medidas generales: espesamiento de las tomas, posicin semiincor-
porada.
Medidas farmacolgicas: procinticos 15-20 minutos antes de tres to-
mas, anticidos o inhibidores de la bomba de protones si existe eso-
fagitis.
Ciruga: cuando no se observa mejora con el tratamiento mdico o
aparecen complicaciones severas del RGE se procede a realizar una
funduplicatura de Nissen (respuesta 5 incorrecta).
P186 MIR 2001-2002
La invaginacin supone la causa ms frecuente de obstruccin intestinal
entre los tres meses y seis aos de edad. Se produce cuando un segmento
intestinal se introduce en otro segmento inmediatamente distal a l. La
forma ms usual de invaginacin es la ileoclica y la ileoileoclica.
La gran mayora de los casos son de etiologa desconocida. Un pequeo
porcentaje son secundarios a procesos como las infecciones por adenovi-
rus, divertculo de Meckel, plipos y otros tumores, etc.
Clnicamente, se manifesta con dolor abdominal intenso de tipo clico,
encogimiento de miembros inferiores y palidez cutnea. En fases inicia-
les suelen aparecer vmitos. En las primeras horas de evolucin puede
haber eliminacin de heces, pese a que si la invaginacin progresa cesa
la evacuacin de gases y heces. Hasta un 60% de los nios afectados
pueden presentar heces con sangre roja fresca y moco ("heces en jalea
de grosella"). Si progresa, el nio puede entrar en un estado parecido
al shock.
El diagnstico se establece por la clnica y exploracin fsica (se palpa una
masa alargada, dolorosa, localizada en hipocondrio-fanco derecho). En el
enema opaco se observa un defecto de replecin al nivel de la cabeza de la
invaginacin, y en la radiografa simple de abdomen, un rea de aumento
de densidad en hemiabdomen derecho y de distensin de asas en el iz-
quierdo. La prueba fundamental es la ecografa, donde es tpica la imagen
en rosquilla o donut.
El tratamiento consiste en la reduccin hidrosttica o neumtica bajo
control ecogrfco o fuoroscpico. Es preferible la correccin quirrgica
en caso de signos de perforacin intestinal, shock, neumatosis intestinal o
distensin abdominal de ms de 48 horas de evolucin.
T5
Nefrologa y urologa
P151 MIR 2010-2011
Caso clnico en el que nos presentan una nia de 10 aos con exploracin
fsica, talla y TA normales, el anlisis de orina tiene una densidad y un pH
dentro de los valores normales, el nico dato alterado es el nmero de he-
mates por campo (normal 0-4).
Puesto que no tiene ningn sntoma (anlisis de orina de rutina), crece
correctamente y no nos indican ninguna otra alteracin en el sedimento,
carecen de sentido las opciones 1, 2, 3 y 5, por tanto, la conducta inicial
ms adecuada sera repetir el sedimento en 15 das, ya que el aumento de
hemates podra ser debido, por ejemplo, a la realizacin de ejercicio fsico
intenso, algo muy comn en esta edad.
P189 MIR 2007-2008
Hasta la edad de los dos o tres aos, los lactantes presentan una fmosis
fsiolgica en un altsimo porcentaje de casos (respuesta 1 correcta). Por
ello, es evidente que no todos debern ser intervenidos quirrgicamente.
Las indicaciones para la circuncisin en menores de un ao son:
Que la fmosis produzca infecciones urinarias.
Balanopostitis.
Parafmosis de repeticin.
Fimosis puntiforme (la que deja un orifcio prepucial mnimo, de forma
que obstaculiza la libre emisin de orina).
Es cierto que los corticoides pueden ser tiles en algunos casos y evitar
la circuncisin, pero esto no puede entenderse como la regla general, de
manera que la respuesta 5 es falsa.
P193 MIR 2002-2003
Se trata de una pregunta de difcultad alta sobre un tema que puede apa-
recer tanto en Pediatra como en Nefrologa o Infecciosas.
El sndrome hemoltico urmico, desde el punto de vista peditrico, es la
causa ms frecuente de insufciencia renal aguda en nios menores de
cuatro aos. Desde el punto de vista infeccioso destaca que casi siempre
va precedido por una gastroenteritis enteroinvasiva por E. coli (si bien tam-
bin se ha relacionado con Shigella, Salmonella y Campylobacter). Desde el
punto de vista clnico la trada tpica: anemia hemoltica con esquistocitos,
trombopenia con pruebas de coagulacin normales e insufciencia renal.
El principal diagnstico diferencial se establece con la trombosis de la vena
renal que cursa con HTA + hematuria + trombopenia.
El tratamiento mdico de las manifestaciones hematolgicas, junto con
una dilisis peritoneal precoz, es el que mejores expectativas de recupera-
cin renal ofrece, de tal manera que la funcin renal se recupera en el 90%
de los casos y es una enfermedad que no recidiva.
De esta manera las pistas que podemos encontrar en este caso clnico son:
Cuadro febril con diarrea mucosanguinolenta.
Orina hematrica.
Anemia.
Trombopenia con normalidad de las pruebas de coagulacin.
Podemos descartar otras opciones como:
Prpura de Schnlein-Henoch: la simple presencia de trombopenia
descarta esta opcin, porque es una prpura no trombopnica. Su
manifestacin cutnea no son las petequias sino la prpura palpa-
ble. Igualmente suele asociar glomerulonefritis, artralgias y dolor
abdominal.
Glomerulonefritis aguda postinfecciosa: suele aparecer en relacin
con infeccin farngea o cutnea por estreptococo del grupo A tras
un perodo de latencia, no coincidiendo con la infeccin. Desde el
punto de vista renal cursa en forma de sndrome nefrtico (hematu-
ria, HTA y oliguria).
515
Desgloses comentados
Pediatra
P198 MIR 2002-2003
Pregunta fcil sobre la infeccin del tracto urinario.
Las caractersticas de la ITU en nios son muy similares a las del adulto. En ge-
neral, es ms genrico en el sexo femenino, la nica excepcin es el primer ao
de vida, siendo ms frecuente en este caso en los varones (opcin 1 incorrecta).
La causa ms usual de ITU es E. coli (opcin 2 correcta). Tambin podemos
encontrar otros gramnegativos como Klebsiella, Proteus... Menos frecuen-
tes son los grampositivos (enterococo, S. epidermidis), adenovirus (cistitis
hemorrgica), hongos...
La clnica en los nios es bastante inespecfca. Lo que nos puede dife-
renciar una ITU baja de una alta es la presencia de febre en esta ltima.
Esta distincin es primordial, sobre todo a la hora del tratamiento: baja-
antispticos urinarios, trimetropn-sulfametoxazol o amoxicilina oral; alta-
aminoglucsidos, cefalosporinas de 3 generacin o amoxiclavulnico ini-
cialmente por va sistmica (opcin 4 correcta).
El diagnstico defnitivo nos lo da el cultivo. En cuanto a la evaluacin
posterior, la ecografa abdominal se indica siempre ante cualquier ITU
en un nio (opcin 3 correcta), mientras que la combinacin de eco, cis-
touretrografa miccional (despistaje de refujo vesicoureteral y disiner-
gia vesical) y gammagrafa renal con DMSA (mtodo ms sensible para
demostrar presencia de cicatrices) (opcin 5 correcta) estara indicada
ante toda ITU febril, ITU en menores de cinco aos o en nias en edades
posteriores que hayan presentado dos o ms ITU.
P187 MIR 2001-2002
El caso clnico nos muestra un cuadro de escroto agudo. La torsin testicu-
lar es la causa ms comn de escroto agudo en menores de seis aos, y el
primer cuadro a considerar en la edad prepuberal o en el adolescente con
dolor escrotal agudo y tumefaccin testicular.
Clnicamente, la torsin testicular cursa con dolor agudo y tumefac-
cin del escroto, exigiendo diagnstico diferencial con la orquiepidi-
dimitis.
Tambin hay que hacer el diagnstico diferencial con la torsin del
hidtide. sta es ms frecuente entre los siete y los 12 aos, suele pro-
ducir menor dolor y tumefaccin, y ocasionalmente se observa una
zona azulada por encima del testculo con sensibilidad aumentada a
la palpacin.
El tratamiento de la torsin testicular es la ciruga inmediata (respuesta 5
correcta). El procedimiento depende del tiempo de evolucin. Si lleva me-
nos de seis horas, se procede a la detorsin y fjacin al escroto. Despus
de seis horas suele ser necesaria la orquiectoma, realizndose tambin la
fjacin del testculo contralateral.
Repasa el diagnstico diferencial del escroto agudo con la tabla correspon-
diente a esta pregunta.
P188 MIR 2001-2002
El refujo vesicoureteral se produce como consecuencia de un fallo en el me-
canismo valvular que impide el paso de orina desde la vejiga hacia el urter.
El dao renal viene determinado tanto por las altas presiones vesicales que
se transmiten a los clices renales como por la frecuencia de infecciones,
que conducirn fnalmente a la existencia de cicatrices renales.
El refujo se clasifca desde el punto de vista etiopatognico en primario o
secundario a diversas patologas tales como duplicacin ureteral, divert-
culo, ureterocele, etc.
Existe una clasifcacin en cinco grados, segn la altura que alcanza el re-
fujo y el grado de dilatacin pieloureteral, que tiene implicaciones prons-
ticas y teraputicas.
P187 (MIR 01-02) Diagnstico diferencial del escroto agudo
516 Desgloses comentados
Pediatra
Una vez diagnosticado, graduado y establecido si el refujo es primario
o secundario, conviene conocer el tamao de los riones y si muestran
cicatrices de esclerosis, que se pueden apreciar ya a los cuatro o cinco
meses de haber padecido una infeccin urinaria (respuesta 3 correcta); la
ecografa permite valorar el tamao renal, y la gammagrafa isotpica del
parnquima renal es til para descartar los tractos cicatrizales. La urogra-
fa i.v. y la tomografa tambin pueden cumplir estos propsitos.
Finalmente, se medir la presin arterial, as como el aclaramiento ba-
sal de creatinina.
El tratamiento del refujo primario difere segn el grado:
En los grados I y II se observa una elevada probabilidad de remisin
espontnea. El tratamiento ser expectante, con proflaxis antibitica
para mantener estril la orina, y seguimiento trimestral con uroculti-
vo. La ecografa renal anual o bienal tambin resulta til para valorar el
crecimiento renal. La gammagrafa renal con DMSA aplicada al concluir
el perodo de tratamiento ayuda a seleccionar a los nios con tractos
cicatrizales residuales que requieren seguimiento de la presin arterial.
El seguimiento de los casos de refujo grado III es muy parecido, pero
se precisan gammagrafas peridicas del parnquima renal si se sos-
pecha la formacin de nuevas cicatrices. Ms de la mitad de los nios
con refujo grado III acaba precisando ciruga.
El refujo grado IV y V requiere ciruga precoz, pues no es la norma la
remisin espontnea.
El tratamiento del refujo secundario es el de la causa.
Los resultados tras un tratamiento quirrgico son excelentes.
P188 (MIR 01-02) Grados del reflujo vesicoureteral
P188 (MIR 01-02) Clasificacin del reflujo
T6
Hemato-oncologa peditrica
P187 MIR 2005-2006
Pregunta de difcultad media acerca del neuroblastoma.
El neuroblastoma es un tumor del sistema nervioso simptico que cons-
tituye el 10% de los tumores de la infancia. La patologa oncolgica ms
frecuente en la infancia son las leucemias representando el 30%. El tumor
slido ms corriente es el tumor cerebral, y dentro de ellos, el astrocitoma.
El tumor slido extracraneal ms frecuente es el neuroblastoma.
Te ayudar saber el orden de frecuencia (de mayor a menos) de los cn-
ceres en el nio: leucemias, cerebrales, linfomas, neuroblastoma, Wilms,
tumores seos, otros (retinoblastoma...).
La edad media al diagnstico son los dos aos y el 90% son menores de
cinco aos (nuestra pregunta nos habla de un lactante de diez meses). La
localizacin ms comn es el abdomen (70%), siendo el 50% de localiza-
cin suprarrenal.
P187 (MIR 05-06) Localizacin del neuroblastoma
La clnica depende de la localizacin. En la pregunta nos dan el caso tpico
de localizacin abdominal que es la ms frecuente y que suele aparecer
como una masa abdominal que sobrepasa la lnea media (a diferencia
del Wilms). Otras formas de presentacin son:
517
Desgloses comentados
Pediatra
Paravertebral: con clnica de compresin medular.
Trax: se suelen descubrir al hacer Rx de trax por otras razones.
Cabeza y cuello: pueden dar un sndrome de Horner.
Nasofaringe (estesioneuroblastoma): suele manifestarse por epistaxis.
La localizacin ms usual de las metstasis es por va linftica y hemtica
a hgado, mdula sea y esqueleto. Consecuentemente, la opcin 3 es
correcta, puesto que ante todo diagnstico de neuroblastoma se debe rea-
lizar un aspirado de mdula sea como parte del estudio de extensin al
ser la mdula una diana frecuente de metstasis.
Para el diagnstico se pueden usar:
TC abdominal que visualiza el tumor como una masa de densidad
mixta slida y qustica, adems de calcifcaciones en el 80%.
Catecolaminas urinarias elevadas (cido homovalnico, cido vanil-
mandlico): ste es un dato especfco de este tumor.
Gammagrafa con MIBG (metayodobencilguanidina) marcada con
istopo radiactivo que capta catecolaminas.
Por consiguiente, la opcin 5 es falsa, dado que esta tcnica se sigue
usando.
Biopsia, que permite el diagnstico anatomopatolgico.
Los factores pronsticos, si bien difciles de retener, son esenciales de cara
al MIR, puesto que ya han sido objeto de preguntas en otras convocatorias.
De buen pronstico:
- Diagnstico en menores de un ao: por tanto, opcin 1 falsa.
- Localizacin en cuello, mediastino posterior o pelvis.
- Vanilmandlico/Homovalnico mayor de 1.
- Aneuploida del ADN (este aspecto ya fue preguntado en exme-
nes anteriores).
De mal pronstico:
- Localizacin abdominal.
- Enolasa srica y/o ferritina elevadas.
- Amplifcacin del oncogn n-myc.
- Estadios III y IV.
- Delecin del brazo corto del cromosoma 1.
El tratamiento debe ser quirrgico siempre que sea posible, si es
necesario administrando previamente quimioterapia en los tumores
no resecables. En consecuencia, la opcin 2 es falsa, puesto que la
presencia de metstasis hepticas no contraindica el tratamiento
quirrgico.
Por ltimo, sealar que la hemihipertrofa es una malformacin cong-
nita que se asocia al tumor de Wilms o nefroblastoma y no al neuro-
blastoma.
P189 MIR 2001-2002
El tumor de Wilms es el tumor abdominal ms frecuente en la infancia y el
tumor renal ms habitual. Puede presentarse aislado o asociado a diversas
malformaciones congnitas.
Las ms usuales son las anomalas genitourinarias, seguidas de la hemi-
hipertrofa y la aniridia. Se ha asociado a algunos sndromes: sndrome
WARG (Wilms, aniridia, retraso mental y malformaciones genitourinarias),
sndrome de Dennys Drash (Wilms, retinopata y alteraciones genitourina-
rias) y el sndrome de Beckwith-Wiedemann. Se pueden encontrar delecio-
nes del cromosoma 11.
La edad media de diagnstico de este tumor es hacia los tres aos de edad
(la edad media de diagnstico del neuroblastoma es algo menor, hacia los
dos aos).
La forma ms frecuente de debut es como una masa abdominal asin-
tomtica (tambin es la forma ms comn de presentacin del neuro-
blastoma).
Otras manifestaciones clnicas son la hipertensin arterial (por com-
presin de la arteria renal por el tumor), hematuria, policitemia por
produccin de eritropoyetina y hemorragia intratumoral por traumatis-
mos. La localizacin ms frecuente de metstasis es el pulmn.
La TC muestra una masa poco homognea, con zonas de necrosis, hemo-
rragia y calcifcaciones focales pequeas (aunque menos corrientes y me-
nos acentuadas que en el neuroblastoma).
Si bien la PAAF no ha demostrado aumentar el riesgo de diseminacin del
tumor en el trayecto de la puncin, la biopsia no debe hacerse porque la
rotura de la cpsula renal cambia el estadio.
El tratamiento se establece con ciruga, radioterapia y/o quimioterapia, de-
pendiendo del estadio.
Te resultar ms fcil estudiar el neuroblastoma y el nefroblastoma si los
comparas. Para ello, puedes utilizar la tabla de la pgina anterior.
P189 (MIR 01-02) Neuroblastoma frente a nefroblastoma
518 Desgloses comentados
Pediatra
T7
Enfermedades infecciosas
P231 MIR 2011-2012
En esta pregunta se plantea el diagnstico diferencial de enfermedades
que cursan con febre y exantema. En este caso clnico debes buscar las
pistas que te lleven al diagnstico. Lo primero que debe llamar tu atencin
son las lesiones blanquecinas en cavidad oral, que no son otras que las fa-
mosas manchas de Koplik, que son patognomnicas de sarampin, siendo
sta la respuesta correcta. Recuerda que, el sarampin se presenta como
un exantema morbiliforme que no blanquea a la presin y puede afectar
a palmas y plantas, en el contexto de una infeccin respiratoria con febre
alta. El hecho de que el nio tenga 7 meses apoya ms el diagnstico, ya
que an no ha recibido la vacuna de la triple vrica (cuya primera dosis se
administra entre los 12 y los15 meses).
Es importante tambin saber por qu no son las otras opciones. En primer
lugar, la enfermedad de Kawasaki se diagnostica mediante criterios clni-
cos fciles de recordar con la regla del ABCDEF. La F de Fiebre de al menos
5 das es el nico criterio imprescindible y debe acompaarse de al menos
4 de los otros 5:
A de Adenopata (cervical mayor de 1,5 cm).
B de Boca (afectacin de labios, lengua y orofaringe).
C de Conjuntivitis (bilateral no exudativa).
D de Dedos (eritema, edema y posterior descamacin de manos y pies).
E de Exantema.
Recuerda que estos sntomas no se pueden justifcar por ninguna otra causa.
En segundo lugar, no puede tratarse de una escarlatina aunque presente
una faringoamigdalitis, ya que en menores de 3 aos stas son prcticamen-
te en su totalidad vricas y recuerda que la causa de la escarlatina es un Strep-
toccocus pyogenes productor de toxina eritrgena. Fjate adems en que no
ha respondido al tratamiento antibitico. En relacin al exantema alrgico
por amoxicilina, se presentara como una reaccin de tipo urticarial. Por l-
timo, podras pensar que es una mononucleosis por el exantema asociado
al empleo de amoxicilina, que es muy caracterstico. No obstante, deberan
aportar otros datos como adenopatas mltiples o hepatoesplenomegalia.
P172 MIR 2009-2010
Este caso sugiere un dolor abdominal recidivante de origen funcional (no
orgnico), que es frecuente en la infancia. Tratados de Pediatra, como el de
Nelson, sostienen que el estudio de laboratorio no siempre es necesario en
una primera visita. Sin embargo, tambin admite como manejo diagnsti-
co inicial la realizacin de un hemograma completo, con VSG, parsitos en
heces y anlisis de orina. Por ello, dado que son posibles varias actitudes,
se tratara de una pregunta dudosa.
Por otra parte, los protocolos de la Asociacin Espaola de Pediatra sostie-
nen que, ante un cuadro de dolor abdominal recurrente debemos cumplir
ciertas condiciones para descartar un origen orgnico. Se deben descartar
los siguientes signos de alarma:
Clnicos (anamnesis y exploracin).
Por datos de laboratorio (alteraciones en el hemograma, marcadores
infamatorios elevados, anomalas del sedimento urinario y presencia
de sangre oculta en heces).
De acuerdo con los protocolos vigentes, sin la realizacin de pruebas com-
plementarias no quedara descartado el origen orgnico, por lo que la pre-
gunta se anul.
P186 MIR 2006-2007
Con respecto de las vas de transmisin vertical del VIH neonatal, recuerda
que la tasa de transmisin oscila entre 25-52% y que esta transmisin pue-
de acontecer en tres momentos: va transplacentaria (30-40% de los casos).
Va de transmisin durante el parto: va ms importante de transmisin
maternofetal (60-70%).
Dentro de las variables que facilitan la transmisin destaca la rotura de
membranas superior a cuatro horas; otras son la prematuridad y el bajo
peso al nacimiento.
Va postnatal: existe documentacin de transmisin mediante la leche ma-
terna (es una de las pocas contraindicaciones de la lactancia materna, salvo
en los pases en vas de desarrollo).
P185 MIR 2005-2006
Pregunta fcil en forma de caso clnico tpico de la enfermedad de Kawasaki.
Esta pregunta expone, en forma de caso clnico, la historia de un pa-
ciente con enfermedad de Kawasaki. Esta entidad se diagnostica me-
diante el cumplimiento de sus criterios diagnsticos. Adems de la
fiebre, aqu aparecen cuatro: exantema, conjuntivitis bilateral no puru-
lenta, cambios en la mucosa oral (eritema labial, lengua aframbuesada)
y adenopatas.
La enfermedad acostumbra a presentarse con febre alta de ms de cinco
das de evolucin que no responde a antibiticos (el nio ha recibido tres
dosis de azitromicina).
Adems de los cuatro criterios diagnsticos que aparecen en la pregunta,
pueden aparecer alteraciones en las zonas perifricas de las extremidades,
como edema y eritema en manos o pies, descamacin de inicio periun-
gueal, que suelen aparecer entre la primera y la tercera semana. La mani-
festacin ms signifcativa por su gravedad es la afectacin cardaca, que
ocurre en un 10-40% de los casos.
Se produce vasculitis coronaria en las dos primeras semanas con posterior
formacin de aneurismas en cuentas de rosario.
Una regla mnemotcnica acerca de las caractersticas de la enfermedad
de Kawasaki: ABCDEFG: adenopatas, boca (alteraciones mucosa orofarin-
ge), conjuntivitis, descamacin, exantema, febre y gammaglobulina en su
tratamiento.
Recuerda un dato muy novedoso (ao 2005) acerca de la etiologa de dicha
enfermedad: se relaciona con el coronavirus New Haven que actuara como
un superantgeno.
La clnica del exantema sbito es muy tpica y fcil de reconocer en el
MIR. Se produce en nios menores de tres aos. Se caracteriza por fie-
bre muy alta con aparente buen estado general y sin foco durante tres
a cinco das y al tercer o cuarto da la desaparicin brusca de la fiebre
coincidiendo con la aparicin de un exantema que no afecta ni a cara
ni a extremidades inferiores. Lo causa el VHH tipo 6.
519
Desgloses comentados
Pediatra
La escarlatina es un cuadro brusco con febre y aspecto de enfermedad
grave con una lengua en fresa blanca (recubierta por una capa blanque-
cina) primero y una lengua roja (por descamacin aparece una lengua
hipermica). Las amgdalas aparecen cubiertas de un exudado blanco
acompaadas de un enantema en el paladar blando. Asimismo, aparece
un exantema que predomina en pliegues y que se palpa mejor que se ve
(lneas de Pastia). En la cara existe un intenso eritema en frente y mejillas
que respeta el tringulo nasogeniano (facies de Filatov). El cuadro est pro-
ducido por el S. pyogenes.
La rubola se caracteriza por una fase prodrmica de catarro leve con
febre moderada y conjuntivitis sin fotofobia. El signo ms caracterstico
es la aparicin 24 h antes del exantema de adenopatas retroauriculares,
cervicales posteriores y postoccipitales. El exantema es morbiliforme,
se inicia en zonas retroauriculares y desde all desciende centrfugamen-
te afectando en 24 h a todo el cuerpo, respetando palmas y plantas.
La mononucleosis infecciosa afecta tpicamente a sujetos entre 15 y 25
aos y la produce el VEB.
El cuadro comienza con sntomas gripales que duran 7-14 das seguidos de
un cuadro forido con febre alta, dolor farngeo intenso, adenopatas de
predomino cervical, hepatoesplenomegalia y rash cutneo maculopapular
(sobre todo, en pacientes tratados errneamente con ampicilina al supo-
ner que el cuadro de faringitis es de etiologa bacteriana).
P185 (MIR 05-06) Enfermedades exantemticas y afines
520 Desgloses comentados
Pediatra
P187 MIR 2004-2005
Las enfermedades exantemticas son un tema relativamente importan-
te. Lo esencial es saber hacer diagnstico diferencial entre ellas. Este
caso clnico presenta datos muy tpicos, con lo que el diagnstico no
ofrece duda.
Fiebre alta + exantema cuando sta desaparece = EXANTEMA SBITO
Es muy tpico de esta enfermedad que el exantema respete la cara y ex-
tremidades inferiores. Recuerda que el agente causal es el HHV-6. Tam-
bin es caracterstico de esta enfermedad la presencia de leucocitosis
con neutroflia en las primeras 24 horas y su asociacin con las convul-
siones febriles.
P172 MIR 2003-2004
Se trata de una pregunta fcil que se responde con informacin del Manual
CTO.
La enfermedad de Kawasaki o sndrome mucocutneo ganglionar es
de causa desconocida, pese a que se postula la hiptesis de una toxina
que inducira un dao inmunitario endotelial (respuesta 3 verdadera).
La enfermedad cursa con fiebre alta de ms de cinco das de evolucin,
con mala respuesta a antibiticos (respuesta 2 verdadera) conjuntivitis
bilateral, afectacin de mucosa orofarngea y de zonas perifricas de
extremidades, fundamentalmente descamacin de inicio periungueal,
exantema polimorfo, linfadenopatas cervicales, etc. El diagnstico es,
fundamentalmente, clnico (respuesta 1 falsa).
La complicacin ms grave es la afectacin coronaria en las dos primeras
semanas con aparicin de aneurismas "en cuentas de rosario" (respues-
ta 4 verdadera) El tratamiento fundamental es gammaglobulina i.v. que
previene la afectacin coronaria y salicilatos a dosis altas (respuesta 5
verdadera).
T8
Sndrome de la muerte sbita
del lactante (SMSL)
P181 MIR 2006-2007
Se consideran factores de riesgo biolgico para el sndrome de la
muerte sbita del lactante el sexo masculino, los antecedentes de
prematuridad (dos o tres veces ms riesgo), la falta de regulacin
trmica, la postura en prono para dormir, mayor frecuencia cardaca
en todas las fases del sueo-vigilia, defectos autonmicos, patrones
respiratorios con alteraciones en la capacidad de autorreanimacin
con peor recuperacin desde una hipoxemia.
El QT prolongado tambin favorece la muerte sbita.
P166 MIR 2003-2004
Pregunta muy fcil, prcticamente de conocimiento popular. De la muerte
sbita, principalmente, hay que saber que se relaciona con el decbito pro-
no y el tabaquismo materno.
El SMSL se defne como la muerte repentina e inesperada en un lactante
por razones que no quedan claras ni despus de la necropsia. Es la forma
de muerte ms comn en el primer ao de vida. La mxima incidencia
es a los dos o tres meses de edad. Ocurren pocos casos antes de las dos
semanas y despus de los seis meses. Los varones tienen ms riesgo que
las mujeres. La incidencia es mayor en los meses fros. Se ha supuesto que
se produce durante el sueo, pues la mayora de los casos se dan entre la
medianoche y las nueve de la maana. Se ha relacionado con mltiples
factores biolgicos y epidemiolgicos, entre ellos la posicin en decbito
prono y lateral (parece que porque interferen con el control respiratorio,
sobre todo en el momento del despertar).
T9
Maltrato infantil
P259 MIR 2008-2009
Una pregunta muy interesante. En ella, intentan confundirnos con infec-
ciones exantemticas como la varicela o la enfermedad de Kawasaki, pero
si somos observadores es difcil que nos engaen. Se trata de un nio mal-
tratado, y el dato principal lo tenemos en la piel, que nos permite descartar
el resto de las opciones.
Una sepsis con CID producira lesiones diseminadas y malestar gene-
ral. No justifcara una distribucin como la descrita, y todava menos
un cefalohematoma parietal.
La enfermedad de Kawasaki no produce lesiones purpricas, o al me-
nos sera muy infrecuente.
Carece, por otra parte, de otros criterios sugestivos (conjuntivitis, ade-
nopatas, etc.).
El dfcit congnito de antitrombina III produce un estado de trom-
boflia, es decir, tendencia a la hipercoagulabilidad, mientras que aqu
estn describiendo hematomas y equimosis.
No existen lesiones vesiculosas (ni vesiculohemorrgicas) que pudie-
ran hacernos pensar en la opcin 4. Tampoco referen febre, prurito,
etc.
Sin embargo, la caprichosa distribucin de las lesiones y los bordes bien
delimitados nos hacen pensar en una causa externa. Si, adems, el nio
no est bien nutrido y est muy agitado, habra que considerar un posible
maltrato.
T10
Calendario vacunal infantil
P183 MIR 2010-2011
Pregunta de elevada difcultad sobre un tema bastante preguntado en el
examen MIR: las inmunizaciones.
La respuesta 1 es correcta, ya que en estudios recientes se ha conclui-
do que la vacunacin antineumoccica masiva de la poblacin infantil
menor de 2 aos ha reducido signifcativamente la tasa de meningitis y
enfermedad neumoccica invasiva (por los serotipos incluidos en la va-
521
Desgloses comentados
Pediatra
cuna) en el conjunto de la poblacin. Existen resultados convincentes de
que la introduccin de la PCV7 en la poblacin infantil en los EE.UU. ha
reducido la incidencia de enfermedad neumoccica invasiva en los nios
y en los adultos, por lo que sera esperable que esto ocurriera en otros
pases.
La respuesta 2 es falsa, ya que la PCV7 tiene su indicacin en nios me-
nores de 2 aos, no en adultos. Las respuestas 3 y 4 son falsas porque
hacen referencia a la vacuna polisacrida polivalente (PPV23) que se ad-
ministra en nios mayores de 2 aos y adultos con enfermedades cr-
nicas, inmunodepresin, mayores de 65 aos y personas institucionali-
zadas. Tiene una inmunidad poco duradera, por lo que se recomienda
en ciertos casos la revacunacin cada 3-5 aos, y no ha demostrado re-
ducir la incidencia de meningitis y enfermedad neumoccica invasiva.
La mejora en las condiciones de vida y el mejor uso de los antibiticos
(respuesta 5 incorrecta) han influido en la disminucin de la incidencia
de meningitis e infecciones invasivas por neumococo pero sin duda, lo
ms determinante ha sido la vacunacin con PCV7 en nios menores
de 2 aos.
P185 MIR 2010-2011
Pregunta fcil sobre un tema muy repetido en el examen MIR, las con-
traindicaciones generales de las vacunas. La respuesta correcta es la 1
debido a que en cuadros con alteraciones inmunitarias importantes (in-
munodefciencias, tratamiento inmunosupresor intenso y prolongado,
radioterapia) estn contraindicadas las vacunas de microorganismos
vivos atenuados.
Por tanto, de las vacunas que aparecen en el enunciado la nica respues-
ta posible es la triple vrica. El resto son vacunas de toxoides o antgenos
purifcados, por lo que no suponen riesgo de infeccin para un inmunosu-
primido.
Otros temas
P180 MIR 2007-2008
En ocasiones, los nios pequeos se introducen pequeos objetos en la
nariz, en un intento normal de explorar sus propios cuerpos. Lo que se in-
troducen puede ser muy variable: alimentos, semillas, borradores, jugue-
tes. Es un cuadro relativamente frecuente y, slo por la edad del nio, de-
beras haberte planteado seriamente responder la opcin 4. Por otra parte,
la clnica que nos describen encaja bastante bien:
Olor ftido o secrecin nasal sanguinolenta.
Difcultad para respirar por la fosa nasal afectada.
Sensacin de ocupacin en la nariz.
P241 MIR 2005-2006
Pregunta de difcultad media-alta que requiere el conocimiento de patolo-
gas maternas que pueden afectar transitoriamente al recin nacido, por lo
que debers conocerlas si vuelve a caer.
Si nos fjamos bien descubrimos que todas las patologas que nos exponen
tienen una etiopatogenia basada en anticuerpos maternos que pueden
pasar al feto va transplacentaria, radicando la difcultad en conocer en cul
de ellas no aparecen manifestaciones en el recin nacido. En todas pasan
estas inmunoglobulinas, pero en una de ellas, la enfermedad celaca en
concreto, la aparicin de sntomas no solamente depende de la presencia
de stas, sino que requiere otros factores determinantes como el factor ge-
ntico y la presencia de gluten en la dieta (no hay gluten en la dieta de un
recin nacido) y, asimismo, es necesario conocer que en esta enfermedad
existe un intervalo libre de enfermedad que oscila entre tres y seis meses
entre la introduccin del gluten y la aparicin de la enfermedad. Por todo
eso, sta sera la respuesta correcta. Con relacin al resto de opciones que
nos presentan:
Si ahora consideramos la respuesta 1 sera falsa, pues en hasta el
15% de hijos de mujeres enfermas o futuras enfermas de miastenia
gravis aparecen ya en el primer da postparto sntomas como pto-
sis palpebral, debilidad generalizada, difcultades de succin... que
responden bien al tratamiento y que se recuperan, generalmente,
el primer mes.
En el caso del hipertiroidismo primario (respuesta 2) tambin sera
falsa, pues se ha visto como existe paso transplacentario de inmuno-
globulinas estimulantes del tiroides en caso de madres con enferme-
dad de Graves o tiroiditis de Hashimoto (siendo el propiltiouracilo el
tratamiento de eleccin en la embarazada) que causa clnica de tiro-
toxicosis transitoria. Si la madre tom drogas antitiroideas durante la
gestacin, los nios pueden estar asintomticos al nacer y no presen-
tar clnica hasta los cinco o diez das.
Finalmente, las respuestas 3 y 4 son enfermedades ampollosas auto-
inmunes que pueden aparecer en la madre y que en un 5-10% de los
casos los anticuerpos pasan la barrera placentaria y provocan durante
unas semanas ampollas en el recin nacido (intraepidrmicas en el
caso del pnfgo y subepidrmicas en el herpes gestationis), pues estos
anticuerpos ocasionan acantolisis al atacar protenas especfcas en la
piel y en las membranas mucosas.
P188 MIR 2004-2005
Pregunta muy difcil sobre un tema muy concreto de endocrinologa pe-
ditrica. No te preocupes en absoluto si la has fallado.
El raquitismo resistente al calcitriol, tambin llamado tipo II, es de
herencia autosmica recesiva. Se debe a una resistencia perifrica
a las acciones de la vitamina D, en relacin con una anomala de su
receptor. Clnicamente, se manifiesta como un sndrome hipotnico
y con signos y sntomas de raquitismo severo. Caractersticamente, a
diferencia de lo que ocurre en otras carencias de vitamina D, asocia
alopecia total.

You might also like